Vous êtes sur la page 1sur 104

DIRM

MODULE III

RISK MANAGEMENT AND REINSURANCE

Supplementary Study Material

Released by
Committee on Insurance and Pension
The Institute of Chartered Accountants of India
New Delhi

1
CHAPTER 1
ECONOMICS OF INSURANCE
SECTION A

1. Next best use of resource is called:

a. Wisdom
b. Expected cost.
c. Opportunity cost.
d. Marginal cost.
e. None of the above.

2. Economic theory states that prices exactly measure opportunity costs if all markets are ----
------
a. Competitive
b. Imperfect
c. Perfectly competitive
d. Open markets
e. None of the above.

3. One of the following is not essential for competitive insurance market:

a) Economic efficiency is achieved by market competition.


b) Each firm and supplier in a competitive market makes continuous efforts
to innovate.
c) If the market does not suffer from significant imperfections, there will be
no need for government regulations.
d) Not only product and factor markets but also financial markets including
insurance markets suffer from a substantial dose of imperfections.
e) None of the above.

4. Market failure is commonly discussed in terms of these general problems:

a) Market structure.
b) Market information.
c) Public goods.
d) All the above.
e) None of the above.

5. One of the following is an asymmetric information problem:

a) Lemons problem.
b) Age.
c) Sum assured.
d) All the above.

2
e) None of the above.

6. If a few buyers or sellers are able to influence the price of a product / service, we call it…..
a) Information perfection.
b) Price discovery.
c) Market power.
d) All the above.
e) None of the above.

7. These are the ways in which market power arises:

a) Barriers to entry.
b) Product different ion.
c) Price discrimination.
d) All the above
e) None of the above.

8. Insurers in India have to secure license from IRDA to start insurance business. It is an
example of:

a) Market perfection.
b) Market Imperfection.
c) Barriers to Entry
d) All the above.
e) None of the above.

9. Product differentiation in insurance can be offered through:

a) Quality product.
b) After sale service
c) Provision of riders.
d) B&C
e) None of the above.

10. When a single firm can produce a number of products / services at costs lower than those
by multiple firms. It is case of
a) Economies of scale
b) Economies of scope
c) Competitive advantage.
d) All of the above.
e) None of the above.

11. The tendency of individuals to alter their behaviour because of the presence of insurance
is referred to as:

a) Lemons problem.

3
b) Principal-Agent problem.
c) Moral Hazard problem.
d) Info-gap problem.
e) None of the above.

12. One of the following does not form part of ‘Public Goods’;

a) Public education.
b) Public parks.
c) Fire protection services
d) Hotels and Restaurants
e) None of the above.

13. One of the following is not true:

a) Externalities occur when actual costs of producing a good / service are


greater than accounting costs.
b) Externalities occur when a firm’s production or an individual’s
consumption has ‘direct and uncompensated effects’ on others.
c) Positive externality occurs if others benefit; negative externality occurs if
costs are imposed on others.
d) Most environmental problems involve externalities and most of them are
a consequence of shifting costs.
e) None of the above.

14. When a failure of a financial institution leads to failures of other financial institutions in the
market, it is a case of:

a) Market failure.
b) Cascading failures
c) Ripple effect.
d) Orchestrated failure.
e) None of the above.

15. Occurrence of runs in India is normally not possible, because of:

a) Regulators.
b) Monopoly market structure
c) Government intervention.
d) Weak public response.
e) None of the above.

SECTION A -Answers

1.c 2. c 3. e. 4. d. 5. a. 6. c 7. d 8. c. 9. d.10. b. 11. c. 12. d. 13. e. 14. b. 15. b.

4
SECTION B

1. Outline the conditions for perfectly competitive markets.

ANS:

The conditions necessary for a perfectly competitive market (including insurance market) to exist
require perfect and full information, perfect foresight on the part of market participants and
instantaneous market adjustments of prices and quantities. We may discuss the conditions in some
detail.
1. A large number of small, autonomous buyers and sellers so that no single buyer or seller nor
any group of buyers or sellers can influence the market. They are price takers and cannot
influence the price of the product as determined by forces of demand and supply. This
assumption also implies that neither buyers nor sellers collude.
2. There exist no barriers to entry into or exit from the market. If the potential entrants see that
the existing firms in the industry are reaping excess profits, they try to make an entry into the
market. It is not adequate if the existing firms take appropriate action to maintain their position
and profits in the market after the entry of the potential entrants enter the industry. The
existing firm must anticipate the entry of new competitors and take appropriate steps to
prevent entry. Existing firms will set a price that will yield only normal profits, if there is a threat
of new entry. In a competitive market, firms will exit from the industry, if they cannot make
even normal profits or they expect to make greater profits in another industry in the market.
3. Buyers and sellers in the market are well informed about the products. That is, producers
and consumers have all the information that is required for making a decision in regard to
the product or service in question. No one has superior knowledge, which is not available
to another.
4. Producers produce and sell identical products. That is, there exists no differentiation of
products. As a result, buyers have no reason to pay more or less than the market price for
the products of any firm in the industry.

2. “There are four ways in which market power arises” Explain them briefly.

ANS:
(i) Barrier to Entry or Exit
(ii) Product Differentiation
(iii) Price Discrimination
(iv) Economics of Scale or Scope
(i) Barriers to entry or exit :
Suppliers are said to possess market power if there are barriers to entry of new firms into or
exit of existing firms from the industry. Governments often set up such barriers for the
supposed benefit of the public. As barriers to entry become more formidable, the market
power of existing firms increases. Market power means some discretion over price for the

5
firms. Market power may also mean the ability to influence the policies of government and
consumer preferences. An example of government creating barriers to entry. Insurers in
India have to secure a licence from IRDA to enter insurance business and should have a
certain minimum financial capital base besides other regulations.
To maintain their profits or market share, all firms attempt to create barriers to entry of new
firms into an industry. The existing insurance firms might create entry barriers by developing
highly sophisticated expertise in selecting lives for insurance by devoting financial, technical
and human resources.
Creation of barriers to entry to acquire market power by collusive action of insurance
companies is generally viewed with suspicion. Barriers to entry of new firms into insurance
markets are being reduced world over with the spread of the process of liberalization.
(ii) Product Differentiation :
Market power is said to exist in the market if an insurance company can differentiate its
products from those of its competitors in the minds of its customers. The existence of product
differentiation is said to exist if buyers prefer the products of a firm over those of its
competitors. The differences between products need not be real; The customers have to
perceive difference in product quality, convenience, service, market reputation etc., Insurers
all over the world try to differentiate their products from those of other insurers in respect of
provision of riders, after sales service, conferring additional benefits etc.
(iii) Price Discrimination:
In purely competitive markets, all firms charge the same price for identical products. Price
discrimination is said to occur when a firm charges different prices to different groups of
customers for the same product. This will lead to the firm gaining market power and securing
higher than normal profits. Insurance companies may attempt to practice price discrimination
but for the regulatory restrictions imposed by regulatory authorities. When the competition in
insurance markets becomes intense, each insurer tries to maintain or increase its market
share and secure higher profits or maintain its profits by resorting to market segmentation
and charge different prices in each segment.
(iv) Economics of scale:
When a firm’s average cost of production decreases as output increases, the firm is said to
be obtaining scale economics. In such a situation, the larger the firm, the lower is its average
unit cost of production. This works as a barrier to entry of new firms as the new firms cannot
successfully compete with the existing large firms in the market. With market widening and
deepening, there should be even less concern about market imperfections because of scale
economies.
3. Explain the problems of asymmetric information.
ANS:
i) “Lemons” problems
ii) Principal Agent problems

6
iii) Problems of adverse selection
iv) Moral hazard problems
(i) “Lemons” problems: A “lemons” problem occurs when insured knows less than the insurer
about the insurer and its products. Insuredes purchase insurance policies in good faith.
Insurance, especially life and health insurance, is a technical and complicated subject and
very few insured have the knowledge to understand the financial strength of the insurer. A
bulk of insurance regulation is meant to take care of the “lemons” problem for insurance
customers. That is the main reason for Government intervention as well as regulation in the
market and mandating certain disclosures on the part of insurers.
(ii) Principal-Agent Problems: The agent is interested in maximizing his or her personal gain.
This may not be always compatible with the objective of maximizing the gain of the principal.
The principal – agent problems relating to insurance business include the method of
ensuring that the agent does not mis-represent or withhold the necessary information about
the insured and its products from customers or withhold vital information relating to the
customer from the insured.
The principal-agent problems can be to a large extent addressed by providing incentives and
disincentives aimed at aligning the interests of the principal with those of the agent.
(iii) Problems of Adverse Selection: This problem arises when the customer better knows the
customer’s situation than the seller of insurance. The problem of adverse selection is
common in insurance markets all over the world. To fix a fair premium that reflects the
expected value of the losses of the potential insured, the insurer must have full and relevant
information about the insured. The insurance company cannot always be sure that the
potential insured has provided all relevant information. By withholding some vital information
regarding their insurability, some of the insured may be able to obtain favourable prices and
terms and conditions.
(iv) Moral Hazard Problems: The tendency of individuals to alter their behaviour because of the
presence of insurance is referred to as moral hazard problem. Insurers are overly concerned
about the possibility that the beneficiary under a life insurance policy might attempt at
shortening the life of the insured. Another example of moral hazard: Insurance companies
might be concerned about selling a disability income policy under which the insured might
obtain payment that is more than he could earn from employment.

4. What do you understand by Free Rider problems or Public Goods?


ANS:
A purely public good is available to everyone whether they pay for it or not and can be enjoyed by
additional people at no cost to anyone. A classic example of a pure public good that is often cited is
national defence. Public benefit from national defence including those who do not pay tax and it
does not cost any more to add “consumers”, whether through births or immigration. Another feature
of the public goods is that the overall opportunity cost of additional users is essentially zero. As the
opportunity cost is zero, from the perspective of the society the price should be zero.

7
As no one can be excluded from the enjoyment of the public good everyone thinks why bother to
pay for it – the problem of a free rider. Most public goods suffer this free rider problem. If some
beneficiaries do not pay for the good, a producer in the private sector will not earn adequate
revenue to cover production costs i.e. the producer incurs losses and hence the production activity
of that good in the economy may be hampered. If the supply of the good is to be ensured to the
consumers, the government will have to produce that good. Governments generally provide public
goods, financing them from societal revenues (mainly taxes). The existence of a tax system is
justified to an extent by the existence of public goods.

While “pure” public goods are not all that common, but many goods have some “public goodness”
to them. Examples of public goods include public education, lighthouses, police, public parks,
public libraries, fire protection services, and basic research and development.
Insurance industry is characterized by the existence of free rider problems. For example, when
an association of insurance companies lobbies for favorable legislation, all insurance
companies – whether members of the association or not – may benefit. Insurance related free
– rider problems can occur when individuals know that they will receive free emergency
medical care and so decide not to purchase private health insurance.

5. Attempt a note on Externalities or Social Costs.


ANS:
Externality – the fourth category of market failure – is considered the most appropriate for risk
analysis. Externalities occur when actual costs of producing a good or service are greater than
accounting costs. That is, producers can impose positive and negative “spillover effects” on others.
Most environmental problems involve externalities and most of them are a consequence of shifting
costs.
Externalities occur when a firm’s production or an individual’s consumption has “direct and
uncompensated effects” on others. Positive externality occurs if others benefit; negative externality
occurs if costs are imposed on others.
When externalities are present (such as a manufacturing facility which pollutes environment) direct
costs of production (input costs) do not reflect fully true economic costs of production, because of
imposition of uncompensated costs on the neighborhood. As the producer is able to shift a part of
the true production costs, cots of production are understated, the firm’s product prices are lower
than they should truly be. As a result, the firm will produce output larger than what is optimal or
socially desirable, thereby further contributing to pollution. Thus, where negative externalities are
present in the production of a good or service, too much of the good, or service will be produced or
consumed, the price will be low and too little resources and effort will be devoted to mitigate or
eliminate the externality. In contrast, where positive externalities are present too little of the good or
service will be produced, price will be high in the market and not much effort and resources will be
deployed for the purpose of enhancing the positive externality.
It is observed that life and health insurance industries witness both negative and positive
externalities. One striking example of negative externality existing in insurance relates to fraudulent
claims made by insured which lead to higher premiums for everyone. Black and Skipper cite an

8
extreme example of negative externality sometimes associated with life insurance – the designated
beneficiary committing murder with a view to collecting death claim proceeds.

SECTION C (case studies)

1. Gainful Insurance company has lot of plans for expansion and consolidating it s position in
the market. Right now it is enjoying 10% market share on new premiums and it has
ambitious plans for expansion. However in a market like India wherein there are lots of
problems in terms of capacity, willingness, trust, and understanding needs, it is not that
easy to capture a bigger share in market. With market-oriented policies of the government,
competition has become highly rampant in the market across various segments. Mr
Bharadwaj, has taken over recently as CEO of the company and the Board has lot of
expectations from him. His performance is being closely observed. He is encountering
many problems. He wanted to have new blood in the organization and therefore recruited
some fresh MBAs under campus recruitment. Some of them, armed with facts of economy
and industry and general psyche of the public, opened a big fight and requested for plan of
action. Discuss.
ANS:
Modern day economics makes imperial demands on the businessmen to manage multiple risks
in a business and to do it in a cost effective manner. Real world markets never live up to the
textbook description of perfectly competitive markets. However, competition results in
allocative efficiency of resources of the society, besides optimizing consumer choice and
value.
If a few buyers or sellers are able to influence the price of a product or service, we call it
market power. The allocation of resources in such a market is sub optional. In insurance
markets, market power exists almost exclusively on the side of the sellers. There are four ways
in which market power arises.
(i) Barriers to entry or exit:
An example of government creating barriers to entry. Insurers in India have to secure a
licence from Insurance Regulatory and Development Authority (IRDA) to enter insurance
business and should have a certain minimum financial capital base besides other
regulations. To maintain their profits or market share, all firms attempt to create barriers to
entry of new firms into an industry. The existing insurance firms might create entry barriers
by developing highly sophisticated expertise in selecting lives for insurance by devoting
financial, technical and human resources. Likewise, insurers might develop particular
expertise in certain activities, such as product design and development, investment and
marketing. Mr Bharadwaj should in terms of creating some barriers so that Gainful Insurance
Company’s position becomes somewhat stronger.

(ii) Product Differentiation:

9
Market power is said to exist in the market if an insurance company can differentiate its
products from those of its competitors in the minds of its customers. The existence of product
differentiation is said to exist if buyers prefer the products of a firm over those of its
competitors. The differences between products need not be real; The customers have to
perceive difference in product quality, convenience, service, market reputation etc., Gainful
should also aim bringing out a slew of products duly embedding the variations in products
with that of competitors.
(iii) Price Discrimination:
In purely competitive markets, all firms charge the same price for identical products. Price
discrimination is said to occur when a firm charges different prices to different groups of
customers for the same product. This will lead to the firm gaining market power and securing
higher than normal profits. Gainful Insurance can consider price discrimination duly keeping
in view the regulatory restrictions imposed by IRDA.
Economies of Scope arises when a single firm can produce a number of products or
services at costs lower than those by multiple firms. A firm enjoying economies of scope can
obtain market power. Available empirical evidence suggests that economies of scope exist
for joint production of some insurance lines, whereas there is no clear evidence regarding
economies of joint marketing.
The major psychological problems, which fresh MBAs explained, are real risks. They can be
explained as following’. Problems of asymmetric information that occur when one party to a
transaction has relevant information that the other does not have. Such problems of
asymmetric information are grouped under four heads.
i) “Lemons” problems
ii) Principal Agent problems
iii) Problems of adverse selection
iv) Moral hazard problems.
The field staff, particularly advisors are suffering on account of basic information gaps, mis-
information and dishonesty on the part of some clients.
Mr Bharadwaj explained to the new recruits these are the realities of industry. Under imperfect
market conditions, it is not easy for the free flow of information. Precisely on account of this reason,
some people are making profits. But counseling the prospects and bringing about awareness in
them about how insurance industry can help individuals to manage their personal risk and business
risk, will, to some extent, mitigate the problems. After all, the insurance industry funds are required
for long-term investments like infrastructure of the country.

CHAPTER 2

10
INTRODUCTION TO RISK MANAGEMENT

SECTION A

1. One of the following is not true.

a. Risk is often thought of in terms of chance of loss.


b. The element of surprise is an important element of a situation of uncertainty.
c. The importance of uncertainty arises from its influence on the process of decision-making.
d. Uncertainty is a state of human mind.
e. None of the above.

2. The relative variation of the actual outcome from the anticipated outcome is known as:
a. Risk
b. Uncertainty
c. Deviation.
d. Digression.
e. None of the above.

3. The cause of the loss, is defined as;


a. Uncertainty.
b. Hazard.
c. Peril
d. Risk
e. None of the above.

4. If Verma’s car is damaged in a collision with Valmiki’s car, such collision is-

a. Uncertainty.
b. Hazard.
c. Peril
d. Risk
e. None of the above.

5. Defective wiring in a cinema hall which increases the chance of fire, is an example of-
a. Moral hazard.
b. Peril
c. Risk
d. Physical hazard.
e. None of the above.

6. If somebody is setting fire to an insured go down stocked with inventory, with an intention to
make undue profits, it is case of
a. Morale hazard
b. Moral hazard.
c. Evil mind

11
d. Adverse selection.
e. None of the above.

7. If you are keeping your main door open, so that burglars can make easy entry, it is a case of:
a. Morale hazard
b. Moral hazard.
c. Evil mind
d. Adverse selection.
e. None of the above.

8. Damage to property by flood, earthquake or fire is a case of;

a. Pure risk
b. Speculative risk.
c. Dynamic risk.
d. Static risk.
e. None of the above.

9. Sham purchases 500 RIL shares with intent to make profit. He is taking risk.
a. Pure risk
b. Speculative risk.
c. Dynamic risk.
d. Static risk.
e. None of the above.

10. Insurance companies generally insure-

a. Pure risk
b. Speculative risk.
c. Society risk.
d. A&B.
e. None of the above.

11. Dynamic risks are more closely related to:


a. Pure risk
b. Speculative risk.
c. Dynamic risk.
d. Static risk.
e. None of the above.

12. Risks which affect the whole society like natural catastrophes – are examples of-

a. Dynamic risk.
b. Fundamental risk
c. Particular risk
d. Nature risk
e. None of the above.

12
13. When there is robbery in Horizon bank and it lost Rs 1 crore, it is a case of –
a. Fundamental risk.
b. Particular risk
c. Bank risk
d. Dynamic risk
e. None of the above.

14. When there is a failure of a system, human errors, inadequate procedures and controls, we call
it as ….

a. Objective risk.
b. Operational risk.
c. Human risk.
d. Systems risk.
e. None of the above.,

15. The basis of modern insurance theory is:

a. Law of Large Numbers.


b. Theory of Probability.
c. Utility Theory.
d. All the above
e. None of the above.
SECTION A – Answers

1.e 2. a 3. c. 4. c. 5. d. 6.b 7. a 8. a. 9.b.10.a. 11.b. 12.b. 13. b. 14. b. 15. d.

SECTION B

1.Differentiate between ‘Risk’ and ‘Uncertainty’.

ANS:
We often find the terms “risk” and “uncertainty” used interchangeably. However, a distinction needs
to be drawn between the two. Risk is often thought of in terms of chance (or probability) of loss.
Uncertainty falls into two broad categories. There are those for which the probability of occurrence
is calculable either on a priori grounds or through the statistical analysis of a series of similar
events that have occurred in the past. The remainder do not lend themselves to such
measurement either because their occurrence follows no discernable pattern or because they are
unique events. The importance of uncertainty arises from its influence on the process of decision-
making of individuals, businesses as also society.

While risk is a state of nature, uncertainty is a state of human mind. It is therefore


possible to consider a situation risky if a number of outcomes is possible and the actual outcome
that materializes is not known in advance. Thus, risk is defined as the relative variation of the
actual outcome from the anticipated or expected outcome. For instance, for a manufacturing firm,

13
the development of a new product is risky as the profits from the sale of the product in the market
are uncertain before the actual sale. Likewise, the development of a new drug by a pharmaceutical
company is characterized by risk because of as the range of possible outcomes with regard to the
market reception for the drug.

2. What is hazard? Explain different hazards.

ANS:
A condition that creates the chance of loss or increases the chance of a loss is termed a ‘hazard’.
Three major types of hazards are usually distinguished.
a. Physical hazard
b. Moral hazard and
c. Morale hazard
Physical hazard: A physical condition that heightens the chance of loss is called physical hazard.
A large number of examples of physical hazard from our daily life can be cited, such as defective
electrical wiring in a cinema hall which increases the chance of fire, bad and poorly maintained
roads that increase the chance of motor accidents and defective locking system on the main door
of an apartment that increases the chance of theft.
Moral Hazard : Moral hazard is a condition characterized by defects in the character of an
individual such as dishonesty that increases the frequency of loss or severity of loss or both. Moral
hazard is a common occurrence in insurance and is not easy to control. Examples: making a
fraudulent insurance claim, submitting an insurance claim for an inflated amount and setting fire to
an insured godown stocked with inventory.
With a view to controlling moral hazard, insurers take a number of steps such as careful
underwriting practices, and by including a number of provisions in the insurance policy such as
exclusions, deductibles and riders.
Morale Hazard : Sometimes, a distinction is drawn between moral hazard and morale hazard.
While, as defined earlier, moral hazard refers to a deliberate dishonesty resulting in increasing the
frequency or severity of loss, morale hazard refers to carelessness or indifference to loss because
of the presence of insurance. Examples include leaving the main door of a house open to make
entry of a burglar easy, leaving car keys in an unlocked car door, and carelessness in regard to
maintenance of health because of existence of a health insurance policy. Such careless acts
increase the chance of loss.
3. Differentiate between pure risk and speculative risk.

ANS:

A traditional classification of risk distinguishes between pure risk and speculative risk. Pure risk
exists when a situation is characterized by uncertainty as to whether or not loss will occur. Pure
risk does not admit the possibility of gain but only potentiality for loss. Examples of pure risk
include prospect of untimely and premature death, likely damage to property by flood, earthquake,
lightning and fire and catastrophic medical expenses. Speculative risk is present when an event

14
can result in either a gain or a loss or status quo. Examples of situations involving speculative risk
include individual’s decisions to buy shares or investment decisions of business firms or business
ventures and investing in real estate.
Three prime reasons warrant the distinction to be drawn between pure and speculative risks. While
insurance companies basically insure pure risks, speculative risks are generally not considered
insurable, barring a few exceptions like institutional portfolio investments.
Second, while the law of large numbers can be easily applied to pure risks, speculative risks are
not easily enable to the application of law of large numbers which facilitates prediction of future
loss experience by insurance companies. A notable exception is the efficient manner in which
casino operators apply the law of large numbers to the speculative risk of gambling.
Third, while the society is harmed by the presence of pure risk when a loss occurs, society may
benefit despite the occurrence of loss from a speculative risk. There is no doubt that the society
does not benefit from the loss arising from a pure risk situation. A company developing a new
technology to produce computers at a lower cost may benefit the society, as a whole while some
existing computer companies may become bankrupt because of this development, is an example
in this regard.
However, it is possible that in some situations both pure and speculative risks may exist. Likewise,
some of the speculative risk decisions, which are motivated by earning profit, might have an impact
on pure risk exposures. For example, developing and introducing a new product into the market by
a manufacturing firm mainly entails speculative risk. In addition, the decision might also lead to a
pure risk exposure such as potential product liability.

4. Explain dynamic and static risks, Fundamental and Particular risks.


ANS:
Dynamic risks arise from the changes that take place in every society, that is, economic, social,
technological, environmental, and political changes. Static risks are those that would exist in the
absence of such changes.
Dynamic risks are closely related to the speculative risks whereas most pure risks are examples of
static risks.
Fundamental and Particular Risks

Fundamental risks are those which affect the whole of society or a major part thereof, such as
uncertainties arising out of the economic or political system or natural catastrophes such as
earthquakes and floods. They are impersonal in both cause and effect. Particular risks on the other
hand affect mainly the individual or firm and arise from factors over which he may exert some
control. This distinction assumes importance as a social insurance and government insurance
programmes or government subsidies may be necessary to deal with situations of fundamental
risk. A particular risk is a risk that does not affects the entire community or groups of persons within
the economy but affects only individuals.
Examples such as bank robberies and car thefts affect the particular individuals or firms
experiencing such losses. In contrast, a fundamental risk is a risk that affects a group of persons,

15
or the entire economy. Risks such as natural disasters, war, high inflation and cyclical
unemployment are some of the examples of fundamental risk. The recent Gujarat earthquake is
another example point.
5. Explain the following – a) Subjective and Objective Risks
b) Operational Risks
c) Financial Risk
d) Strategic Risk
ANS:
SUBJECTIVE AND OBJECTIVE RISKS
Whether risk is based upon the state of mind of an individual or it is precisely observable -
subjective or objective risk. If the type of risk is more precisely observable and therefore
measurable it is referred to as objective risk. In this type of risk, the actual experience may differ
from the one expected. On the other hand, subjective risk basically emanates from a person’s is
state of mind or mental attitude. As a particular individual’s perception of risk in a particular
situation may be different from another, there is a possibility of making different decisions by two
individuals in a situation that is seemingly identical. It is clear, therefore, that it is not sufficient to
know the degree of objective risk. We have to also learn the attitude towards risk of a person who
is acting on the basis of this knowledge.
OPERATIONAL / INTERNAL RISKS: Operational / internal risks are those that arise from the
operations of an organization or firm the activities of an individual. This type of risk is associated
with failures of a system, human errors, inadequate procedures and controls and deficiencies in
information systems. We may observe that individuals or organizations have some control over
them. Examples of this class of risks include automobile accidents, strike by employees of a firm
and work stoppages, loss of damage to property as a result of fire, uncertainty about legal liability
arising from production of defective or faulty products.
FINANCIAL RISK: Financial Risk arises from individuals or organizations using financial
institutions or ownership of such instruments. Financial risks are those occasioned by changes in
interest rates, transactions involving foreign currency, share issues, extension of business credit
and employment and use of derivative instruments. They are primarily external to the individual or
business. Therefore, this type of risk is not under the direct control of the individual or business.
Individuals making investments, or borrowing funds from a finance company to buy a car or a
residential house or a firm extending credit to its customer are some instance of financial risk.
STRATEGIC RISK : Strategic risks basically arise from economic, demographic, political,
technological and social factors that impact on individuals and businesses. A number of examples
are cited such as consumer preferences, legal system, regulatory environment, terrorism and
global warming. While it is not possible to control these factors and risks involved, it is within the
capability of individuals and businesses to take steps to mitigate the deleterious effects of such
risks.

16
6. Trace out the historical development of the concept of risk.
ANS:
The first notable step in providing a formal and mathematical basis for the theory of probability was
taken in 1654 by two French mathematicians Blaise Pascal and Pierre de Format to resolve a
puzzle that had been teasing mathematicians as well as gamblers for over 150 years. The puzzle
is : How are the stakes between two players in a game of chance is to be divided if the game is
stopped before it comes to an end and one player is ahead of the other. The solution they
suggested was that the two players share the stakes on the basis of the respective probability that
each would win the game. Though the response of Pascal and Fermat was to a puzzle in a game
of chance, their demonstration of the method of calculating the probability of each player’s was a
vital intellectual breakthrough. In the next fifty years after 1654, a number of innovations and
discoveries that are useful as building blocks with the development of tools of risk measurement
were developed such as statistical sampling, statistical significance, various applications of
probability theory to practical problems and early efforts for defining normal distribution and
standard deviation. The three other major components of the science of risk management as we
understand today namely the notion of utility and the concepts of regression to the mean and
diversification have been developed in course of time.
SECTION C (case studies)
1.Mr Sunil Purushotam was awakened by a phone call at 2.30 am on a wintry December night.
Earlier that night the first major storm of the year has hit the coast. Praveen, a risk manager, had
gone to bed unaware of storm’s intensity and was started discovering that a devastation has been
done. The voice on the other end of the line belonged to a police officer, the night patrolling party,
who explained that there is no proper equipment to tackle the problem of storm. Property has been
damaged and many human lives lost besides some livestock which is missing. The district
administration did not anticipate these problems and therefore running short on arrangements.
Were that not bad enough, the officer concluded the call by noting that the storm was really
agonizing. Vehicles that can be used for rescue operations are not available. Some private vehicle
drivers are not cooperating and some of them are drunk and behaving badly. Discuss.
ANS:
This dramatic story unfolds how risk strikes us and how unprepared we are, more so in a country
like India, where risk management as a concept, not gone through the minds of many. We live in a
world of risk and uncertainty. Unexpected events can and do occur and disruptive effects can be
enormous. This example shows both the tangible and intangible consequences of risk – a tragic
loss has occurred, to be sure, but the damage is not limited to physical injury and property
damage. There may be political implications in as much as the incident reflects poorly on
devastation management practices and relief operations. Certainly, the impact on the victims
families is much more than economic. The time the risk manager / district administration spends
tending to this matter could have more productive uses elsewhere. Finally, the resources used to

17
pay for the damages are public resources that otherwise could have been used to carry out any
welfare activity.

2. Over the years, the Olympic games have presented a host of interesting and challenging
exposures to loss. Revenues from the games typically come from the sale of television
broadcasting rights, ticket sales, and commercial sponsorships. Identifying all of the risks
associated with the Olympics is always a major undertaking. In the past, more than one
thousand official vehicles have been required, ranging from vans and buses for shuttling
athletes and personnel to the various events, to ‘stretch’ limousines for transporting visiting
foreign dignitaries. Thus, the collision peril and its associated losses must be recognized
and managed. In planning for the 2000 Summer games in Sydney, Australia, a total of
10,000 athletes were expected, together with about 5000 officials, 50,000 volunteer
workers and 9 million spectators.
Source of risk are carefully analyzed throughout the preparations for each Olympics.
Because many of the sports are inherently dangerous, the possibility of personal injury to
competitors as well as spectators is always present. And the fact that some of the athletes
may be young teenagers only complicates the risk management challenge. The organizers
of the Sydney Games were especially concerned about technological risks (such as
computer malfunctions that might interrupt electronic scoring systems), security (including
the threat of terrorism) and crowd control. Transportation issues in Australia are also
important. Arranging for a governmental guarantee that train schedules would be disrupted
by no more than 5 minutes even if an individual committed suicide on the tracks solved
one interesting risk management problem.
Discuss.
ANS:
Managing public risk of such a magnitude is real challenge. It requires lot of planning and
careful execution. Any glitch on any part can goof up the whole operations and it may
result in a catastrophic loss. Some very serious loss exposures may not be obvious. For
example, weather-related perils can force the postponement of a ticketed event, such as
the opening ceremony. The cancellation of the television coverage contract, often the
largest single source of revenue, would be devastating. Such a cancellation actually did
occur in 1980, when the United States boycotted the Moscow Olympics to protest the
Soviet invasion of Afghanistan.

18
CHAPTER 3

ESSENTIALS OF RISK MANAGEMENT

SECTION – A

1. One of the earliest references to the term ‘risk management’ was made in the US when it
appeared in this magazine;

a. The Harvard Business Review.


b. The Business week.
c. The Forbes magazine.
d. The Life magazine.
e. None of the above.

2. One of the following is not true.

a. The subject of risk management does not have a long history.


b. “ the essence of risk management lies in making the areas where we have some control
over the outcome while minimizing the areas where we have absolutely no control over the
outcome”.
c. The focus of risk management, particularly by corporates, has undergone change over
time.
d. The corporates have become far less concerned with traditional ‘ high frequency, low
severity’ risks
e. None of the above.

3. One of the following is not a step in risk management process.

a. Identify potential losses.


b. Evaluate potential losses.
c. Select the appropriate technique for treating loss exposures.
d. Risk pricing
e. None of the above.

4. The approaches used for risk identification include use of –

a. Loss exposure check lists.


b. Flow charts.
c. Statistical analysis of historical data.
d. Analysis of financial statement.
e. None of the above.

5. According to Rejda, “Accounts Receivables”, can be classified as;

a. Property loss exposure.

19
b. Liability loss exposure.
c. Business income loss exposure.
d. Crime loss exposure.
e. None of the above.

6. According to Rejda, “Kidnapping of key personnel”, can be classified as;

a. Foreign Loss exposure


b. Crime Loss exposure.
c. Human Resources Loss exposure.
d. Business Income loss exposure.
e. None of the above.

7. One of the following is not true.

a. Frequency of loss relates to the probable number of particular losses that may occur
during some given period of time.
b. Severity of loss refers to the probable magnitude of losses that may occur, if they occur.
c. A loss potential that is small even though frequent, is much less important compared to
potential loss exposures that are infrequent but has a potential for bankruptcy.
d. Both frequency and severity have to be considered in the risk management process.
e. None of the above.

8. One of the following is not a technique for treating loss exposures;

a. Hedging.
b. Risk Financing
c. Risk retention.
d. Insurance.
e. None of the above.

9. One of the following is not true;

a. Commercial insurance is a technique of transferring risk from one party to another for a
price.
b. One can combine a large number of independent exposure units in one portfolio; an
insurance company is able to reduce the risk of its aggregate losses.
c. Firms that enter into contracts to supply goods at a fixed price in the future face the risk
that a rise in prices between entering into the contract and the delivery date may involve
them in a loss.
d. Retention refers to the financing of losses internally.
e. None of the above.

10. Insurance is suggested in the following case:

a. Frequency of loss – low and Severity of loss – low


b. Frequency of loss – high and Severity of loss – low

20
c. Frequency of loss – low and Severity of high – low
d. Frequency of loss – high and Severity of loss – high
e. None of the above.

SECTION A –Answers

1.a. 2. d 3. d. 4. e. 5. a. 6. a. 7.e 8. e. 9. e.10. c.

SECTION B

1. Outline the steps of risk management process.

ANS:

STEPS IN RISK MANAGEMENT PROCESS

Identify potential losses.


Evaluate potential losses.
Select appropriate technique for treating loss exposures:
Risk Control – Risk avoidance and Loss Control
Risk Financing – Retention, Non-insurance transfers,
Commercial insurance.
Implement and administer the program.
2. Explain the process of risk identification.
ANS:
The most important element of and the first step in the risk management process is the
identification of risks and exposures to loss. This involves a systematic and careful analysis of all
major and minor potential loss exposures. An essential prerequisite for a conscious choice of
appropriate and efficient methods for dealing with losses if they occur is the recognition of all
sources of possible losses. A loss exposure is a potential loss that may be connected with a
particular category of risk. As such, the classification of loss exposures is the same as the one
adopted for pure risks that is, losses associated with life, health, property and liability risks.
The following categorization of risks that can impinge on the financial security of an individual will
be found useful.
a. Maintenance of a large Emergency Fund :Once it is agreed that prudence demands that a
fund has to be set aside for meeting emergencies, individuals and businesses have to maintain
a larger fund to meet unanticipated loss in the absence of insurance industry.

21
b. Deprivation of certain goods and services: Because of the threat of liability lawsuits, a
number of firms may refrain from producing certain products. It is estimated that out of
approximately 250 firms around the world, which were producing childhood vaccines, only a
few firms now remain in that business.

3. Explain the process of evaluation of potential losses.


ANS:

Subsequent to identification of risk, the second step in the risk management process relates to
analyzing, evaluating and measuring the impact of losses on the individual or corporate unit by
estimating the potential frequency and severity of losses. While frequency of loss relates to the
probable number of particular losses that may occur during some given period of time, severity of
loss refers to the probable magnitude of losses that may occur, if they occur. This will enable
ranking of various loss exposures according to their relative importance. A loss potential that is
small even though frequent, is much less important compared to potential loss exposures that are
infrequent (such as destruction of a factory by devastating fire or accidental deaths) but has a
potential for bankruptcy. Thus, though both frequency and severity have to be considered in the
risk management process, severity of course, is substantially more important. It is possible that a
firm may become bankrupt by a single catastrophic loss – the size of the loss from a single event
can be truly crippling. In the risk management process, it is therefore necessary for estimating
maximum probable loss and maximum possible loss. The maximum probable loss is an estimate of
the worst loss that is likely to happen such as the loss that could likely to result if lightning strikes
an individual’s or home or the firm’s building. The maximum possible loss is an estimate of the
worst possible loss that might result from the lightning.

4. Outline appropriate techniques for treating loss exposures.


ANS:
The techniques are broadly classified as follows:
a. Avoidance: This is the most drastic way of handling the risk. This happens when the activity
causes the risk. For example the potential consequences of an escape of a highly toxic
gas may be so catastrophic that a chemical company may decide to avoid the risk by
ceasing to produce or use it.
b.Risk Financing: This covers all methods employed to fund either the probability of loss–
producing events occurring or the potential size of losses that do occur. For example
installation of fire extinguishers to minimize loss in case of fire. The methods are:
i) Risk Retention : Retention refers to the financing of losses internally – either fully
retained or partially retained. Losses that occur when prior planning for their
financing has been done are also retained. Retention is resorted to when no
other risk management treatment is available like when insurance coverage is
not available or very expensive. Further, non-insurance transfer may be
unavailable. Thus, retention may in fact be a residual method. Retention can
be effectively used when the potential losses are highly predictable.

22
ii) Insurance : Commercial insurance is a technique of transferring risk from
one party (individual or business) for whom the risk is costly to another party
who is willing and is able to bear the risk. Insurance is thus one of a number of
available instruments for hedging risk. It is an instrument for post loss
financing.
iii) Non-Insurance Transfers : They are techniques (other than insurance) by which
a risk exposure and its potential financial losses are transferred to another
party who is in a better position to exercise loss control. A number of instances
of non-insurance transfers can be cited. A computer lease agreement by a
firm may contain a clause to the effect that maintenance, repairs etc., of the
computer are the responsibility of the computer firm. A publishing firm may
specify that the author and not the publisher is legally liable for plagiarism, if
any.
iii) Combination This method takes advantage of the law of large numbers. One
can combine a large number of independent exposure units in one portfolio;
an insurance company is able to reduce the risk of its aggregate losses. The
best example would be in the case of a large group, which by centrally pooling
the risk of breakage of its shop windows could predict annual losses within
narrow limits.
v) Hedging Firms that enter into contracts to supply goods at a fixed price in the
future face the risk that a rise in prices between entering into the contract and
the delivery date may involve them in a loss. Hedging gives protection to the
seller for Indian importers by allowing them to forward purchase specific
foreign currencies. Thus regardless of the exchange rate the importer’s liability
will be limited to the cost of purchasing the currency.
vi) Research: this is done to improve the information on which decisions are
taken. For example when marketing a new product may go for market
research to reduce uncertainty.

23
2. Explain Rajda’s matrix for determining appropriate technique for tackling losses.
ANS:
For the purpose of determining the appropriate technique(s) for tackling losses, Rajda suggests a
matrix. The matrix classifies the various loss exposures on the basis of frequency and severity.
RISK MANAGEMENT MATRIX
Type of Loss Frequency of Severity of Loss Appropriate Risk
Loss Management
Technique
1 Low Low Retention
2 High Low Loss Control and
Retention
3 Low High Insurance
4 High High Avoidance

As can be noticed, the selection of the method for handling losses, both severity and frequency of
loss have to be considered. When the frequency and severity of loss are low, (that is, where the
loss is small and occurs infrequently such as loss of a cheap transistor radio in the house or loss of
the typist’s dictionary in the office) retention is the most appropriate risk management technique.
When the loss frequency is low and severity is high (third type of loss exposure in the table), the
most appropriate technique to choose in insurance. Examples of this type of loss exposure include
natural disasters, explosions, fires, lawsuits and premature deaths.

SECTION C (case studies)


You are the Risk Manager for a large multinational enterprise involved in the manufacture of
products through a hazardous process prone to pollute. Your main operations are in Finland, a
country known for its rigorous and costly pollution, manufacturing and employee safety standards.
Moreover, Finnish workers are highly paid by world standards. One of your employees has
suggested the relocation of the most hazardous aspects of the manufacturing process to a
developing country in Africa. The employee argues that such a move would be sound risk
management because the costs of compliance would be lower in the African country, which has
less rigorous pollution, manufacturing, and worker safety laws. Moreover, because worker wages
would be lower, the costs of worker disability payments and other job-related injuries and illnesses
would be less. Finally, the employee notes that unemployment is rampant in the African country,
and the government and people would welcome the opportunity to work for the company.
As the Risk Manager of the Multinational, your goal is to minimize volatility in corporate cash flow
and to do so at the lowest cost consistent with ethical behavior. If you maintain the hazardous
operations in Finland, costs will remain high. If you relocate them to the developing country, costs
will decrease and people will have more jobs.

24
What is your recommendation? Discuss.

ANS:
The management of risk is a process with the objective of identifying risk exposures faced by an
individual / organization with a view to selecting the best available technique for treating such
exposures. The corporates have become far less concerned with traditional “high frequency, low
severity” risks but have started devoting greater attention to risks that could lead to corporate
collapse and corporate bankruptcy. There is therefore a need to adopt a comprehensive approach
for managing risk. This comprehensive strategy for treating risk is referred to as “integrated risk
management” (IRM) or “enterprise risk management”. (ERM). To achieve the goal and to make the
programme effective, a periodic review of the programme is called in order to determine whether or
not the objectives of the risk programme are being realized.
In the given case, the problem of relocation has to be viewed from risk angle also. What kind of
risks the company has to face when it is located in an African country. What are the local laws? Is
the government there positive to foreign investment? Do they allow repatriation? How are their
pollution laws? Is there any country risk? A Risk Audit has to be conducted to identify potential
sources of risk and then the methods to address to those issues? What will be the financial out flow
involved in accepting any solution? Are there any health issues? Unless these things are
addressed and solutions found, it is premature to suggest to the Board any type of solution.

25
CHAPTER 4

PROBABILITY

SECTION A

1. Probability can take values

(a) - ∝ to ∝ (b) - ∝ to 1 (c) –1 to 1 (d) 0 to 1

2. Probability can expressed as:

a) ratio (b) Proportion (c) percentage (d) all the above

3. When rolling a die the probability that occurring 3 is equal to

(a) ½ (b) 1/3 (c) 1/6 (d) 1/5

4. Mean is a measure of

(a) location (central value)


(b) dispersion
(c) correlation
(d) none

5. Which of the following is measure of central value

(a) Median (b) Standard deviation (c) Mean deviation


(d) Quartile deviation

6. The average of 7 numbers 7,9,12, X, 5,4,11,9. The missing number is

(a) 13 (b) 14 (c) 15 (d) 8

7. Collect formula for variance of ‘n’ sample observations x1, x2…………… xn is:

(a) 1 (b) 1
--- Σ (Xi - X)2 ------ (ΣXi2 - X2 )
n-1 n-1

( c) 1 (d) 1
--- Σ (Xi - X)2 ---- Σ Xi2 - X2
n n

26
8. What is the formula for bionomial distribution?

(a) x!
----------------------- px, qn-x
x! (n-x) !

(b) n!
----------------------- px-1, qn
x! (n-x) !

(c ) n!
----------------------- px, qn-x
x! (n-x) !

(d) n!
----------------------- px, qn
x! (n-x) !

9. The relationship between variance and standard deviation is

(a) variance is the square root of standard deviation


(b) standard deviation is the square of the variance
(c) variance is equal to standard deviation
(d) square of the standard deviation is equal to variance

10. The formula for the co-efficient of variation is

(a) S.D. mean


C.V.= ------ x 100 (b) C.V.= --------- x 100
mean S.D.

(c ) Mean x S.D. 100


C.V. = ---------------- (d) C.V. -------- x S.D.
100 Mean

27
11. The expected value of a set of possible outcomes equal to

(a) E.V. = ∏. Pi.xi (b) E.V. = Σ Pi.xi


( c) E.V. = ∏. Pi.xi2 (b) E.V. = Σ Pi.xi2

12. Give that pi = 0, .05, 0.08, 0.20 & 0.67 and their outcomes ni = 5, 7, 6, & 2 respectively. The
expected value is

(a) 6.35 (b) 4.80 (c) 3.35 (d) 7.29

13.The risk averse utility function is given by

(a) EU = Σ Pi.V (xi) (b) E.U = ∏ Pi. U (xi)


(c ) e.U = Σ Pi.U (xi)2 (d) E.U = Σ Pi.U (xi)

14. Variance of the following frequency of distribution is:

Class Frequency

2-4 2
4-6 5
6-8 4
8-10 1 is approximately
equal to (a) 2.5 (b) 2.9 (c ) 5.0 (d) None

15. The formula for the mean is


xi Σxi
(a) x = ---- (b) x =------
n n

Σ xi m
(c) x = ---- (b) x =------
n Σ xi

SECTION A (Answers)

1. Ans: (d)

28
2. Ans: (d)

3.Ans: (c)

4.Ans: (a)

5.Ans: (a)

6.Ans: (c )

7.Ans:( a)

8.Ans: (c )

9. Ans: (d)

10.Ans: (a )

11.Ans: ((b)

12. Ans: (c )

13. Ans: (d)

14. Ans: (b)

15. Ans: (c )

SECTION B

1. Give briefly the concept of probability?

2. What are the steps included in the calculation of standard deviation?

3. Find mean, median, & mode for the following data


42, 53, 71, 63, 82, 91, 48, 53, 67, 98, 53

4. Write briefly about theoretical probability distributions which are used in risk management?

ANSWERS

1. There are no.of events in day to day life about which one is not sure whether it will occur (or)
not. But one is always curious to know what chance is there for a happening (or) event to occur.
For instance one may be interested to estimate whether it will rain today or not. One would like to
evaluate this chance of winning for head in a definite no.of tosses. What is the chance that there

29
are four aces in one hand in a game of cards among four plays etc. i.e. the numerical evoluation of
chance factor of an event is known as probability.

2. (i) Find the mean


(ii) Subtract the mean from each individual value
(iii) Square the individual differences from the mean
(iv) Total the square of differences
(v) The sum of the squared deviation is divided by the total no.of measurements
(vi) The resulting number is the mean of the squared deviation i.e. variance
(vii)Standard deviation is obtained by taking the square roof of the variance

3. Mean (x ) = Σxi


-----
n

=x 42+53+71+63+82+91+48+53+67+98+53
---------------------------------------------------
11

= 721 = 65.55
11

Median = Arrange in increasing or decreasing order and in which mid value is median

= x Median = 42, 48, 53, 53, 53 63, 67, 71, 82, 91, 98

= 63

Mode = most repeated value

Where mode is 53

3. (i) Bionomial Distribution


(ii) Poisson distribution
(iii) Normal distribution
(i) Bionomial Distribution: (a) It is a discrete probability distribution
(b) the probability that an event will occur at any point in time is P. and the probability
that the event will not occur is given by the eqn, q = 1-P; where no.of ________
as ‘n’ the probability of x successer is ‘n’ trails is given by

P (x – x) = nx px. qn-x

(q) n!
------------- Pxqn-x

30
x! (n-x)!

(ii) Poisson Distribution: The probability of an event under the poisson distribution is given by the
formula.

P = mn. em
-------------

n!

Where ‘p’ is the probability that an event ‘n’ occurs

n – is the no.of events for which is probability estimate is required.


m - is the mean or expected loss frequency
e - is the base of the natural logarithms equal to 2.71828

(iii) Normal distribution

The normal distribution is a continuous probability distribution. It is completely defined by its mean
and standard deviation. The graph of N.D. is bell shaped and symmetric. More values are located
around the mean - the center.
y

O mean x

SECTION C (CASE STUDIES)

1. In a cultivating land the loss occurrence yearly, given in rupees and the probability of loss
occurrence also. Find the mean amount of loss and expected value of loss for 5 years.

Year Loss in rupees Probability of loss

1 6 lakhs 0.05

31
2 28 lakhs 0.18
3 30 lakhs 0.28
4 15 lakhs 0.35
5 58 lakhs 0.14
----------- ------
137 lakhs 1
------------- --------

2. In a cricket team the runs scored between players are given below:

27, 53, 48, 98, 61, 52, 72, 40, 20, 31


what is the variance score between players.

ANSWERS

1. Ans:
Year Loss in rupees Probability of loss Amount of loss
(loss x Prob.of loss)

1 6 lakhs 0.05 0.3


2 28 lakhs 0.18 5.04
3 30 lakhs 0.28 8.4
4 15 lakhs 0.35 5.25
5 58 lakhs 0.14 8.12
----------- ------ ------
137 lakhs 1 27.11
------------- -------- -------

mean amount of loss = 137 = 27.4


5

Expected amount of loss = 27.11

2. Ans:
Scores Mean score Deviation from mean Squared deviation
score
27 55.2 - 28.2 795.24
53 55.2 - 2.2 4.84
48 55.2 - 7.2 51.84
98 55.2 42.8 1331.84
61 55.2 5.8 33.64
52 55.2 - 3.2 10.24
72 55.2 16.8 282.24

32
40 55.2 - 15.2 231.04
20 55.2 - 35.2 1239.04
81 55.2 25.8 665.64
552 0 5145.6

Mean score : 552 = 55.2


10

variance (σ) = 5145.6


------------- = 514.56
10

S.D. (σ) = √514.56 = 22.68

33
CHAPTER -5
DECISION MAKING UNDER CONDITIONS OF UNCERTAINTY

SECTION A

1. According to the Bernoulli, the satisfaction (or) utility derived from marginal
increases in wealth is _________ related to the level of wealth previously possessed.

a). proportionally b). Equally


c). a & b d). Inversely

2. According to the risk averse person:


More wealth is preferred to less wealth; increasing wealth gives
__________levels of satisfaction.

a). decreasing b). increasing


c). equal d). none of the above

3. The premium is equal to the expected value of loss is called________premium.

a).net premium b).net single premium


c).fair premium d).none of the above.

4. The degree of risk aversion of individuals crucially determine the given below
patterns.

a).consumption b). savings


c). aggregate consumption and savings and life insurance
consumption.
d). all of the above.

5. According to Bernoulli the value of a risky opportunity for an individual to become


richer is determined by his (or) her _______ levels of wealth.

a). current b). future expected


c). pre-existing d). none of the above.

6. What is the formula for expected value of an event.

(a) EV= Σ PiXi (b) EU = Σ PiU (Xi)

( c) ∏ Pi Xi (d) None

34
7. Given that P1 = 0.25, P2 = 0.55, P3 = 0.20 and X1 =15,000 , X2 =20,000 and
X3 = 8,000 respectively what is the expected value of this above values.

a).17,350 b). 18,750


c)16,350 d). none of the above

8. If the individual is risk neutral:

. a). he likes risk b)he dislikes risk


c) he is neither averse to risk nor does he like risk
d) none of the above

SECTION A (Answers)
1) d
2) b
3) c
4) d
5) c
6) a
7) c
8) c

SECTION B

1. What is expected utility rule in insurance?


2. Name the various economic theories of consumption and briefly explain.
3. Describe behavioral pattern of human short comings according to Daniel Kahneman and
Amos Tversky?
4. What is the “endowment effect” according to behavioral economists?
5. Name the two characteristics of utility function for a risk averse individual exhibit?
ANSWERS

1. The application of the expected utility rule relates to individuals choice among investment
alternatives. The same rule is also applicable to determine whether an individual will
purchase insurance in order to minimize uncertainty. By purchasing insurance policy, one
pays a premium to avoid a risky outcome. Or purchase of an insurance policy involves the
sacrifice of certain wealth in order to avoid the possibility of a loss of wealth.

2. The various economics theories of consumption are:


a) Absolute income hypothesis
b) Relative income hypothesis
c) Life cycle hypothesis and
d) Permanent income hypothesis

The economic theories of consumption assume that rational consumers seek to maximize their
lifetime utilities while allocating income between current consumption and saving. This

35
assumption implies that individuals divide their income between current consumption and
future consumption in a manner that results in optimizing their pattern of consumption over
lifetime.

3 Behavioral pattern of human short coming according to Daniel Kahneman and Amos Tversky
are:

a) Humans sometimes act on emotion that destroys self – control,


an essential prerequisite for rational decision making
b) People experience what psychologists call “cognitive difficulties”.
In other wards, they often do not understand fully what they are dealing with.

4. Endowment Effect: It describes the tendency of people to set a higher selling price on an
item that they own or endowed with than what they would be willing to pay to purchase an identical
item if they did not own it. The endowment effect has been found to have a powerful influence on
investment decisions that could not be explained by the rational investor model.

5. The two characteristics are:

a) More wealth is preferred to less wealth. Increasing wealth gives increasing


levels of satisfaction
b) The incremental utility or satisfaction from units increases in wealth decreases as
wealth increases. i.e., for a poor person in addition of Rs.200 to his /her wealth
will make a great impact on his / her wealth than the same increase of wealth to a
rich person.

SECTION C (CASE STUDY)

If you invested sum of money 2,00,000 in a private bank that will earn 20% on your investment for
one year with probability of 0.35 and a loss of 15% with probability of 0.65. what is your
expected utility?

ANSWERS:

Given that C = 2,00,000

Earn 20% with probability - 0.35


Loss 15% with probability - 0.65

∴ the expected utility = ΣPi U (Xi)

earn = 2,00,000 X 0.20 = 40,000

36
loss = 2,00,000 X 0.15 = 30,0000

(Xi) = 2,00,000 + 40,000 = 2,40,000


(X2) = 2,00,000 – 30,0000 = 1,70,0000

E.U = 0.35 X U (2,40,000) + 0.65 X U (1,70,000)


= 0.35 X ln (2,40,000) +0.65 X ln (1,70,000)
= 4.3359 + 7.8283
= 12.1642

37
CHAPTER 6
RISK MANAGEMENT PROCESS

SECTION –A

1. The job of risk management can be broken down into following element / s.

a. Risk Assessment
b. Risk Control.
c. Risk Financing
d. All of the above
e. None of the above.

2. …………………comprises of identification and analysis of potential loss exposures

a. Risk Control
b. Risk Dynamics
c. Risk Assessment
d. Risk Initiation.
e. None of the above.

3. Making use of a matrix plotting in loss frequency and loss severity, is known as

a. Risk Analysis
b. Risk Grid
c. Risk Mapping
d. Risk Convergence
e. None of the above.

4. Loss frequency can be categorized under this / these head / s.

a. Rare Loss Event such as a person’s house located in an earthquake zone that is likely to
suffer loss rarely
b. Loss Event that occurs occasionally like a damage to a person’s car
c. Frequent Loss Event such as mild illness
d. All of the above
e. None of the above.

5. One of the following is not true;

a. Financial consequences of relatively cheap utensil of a family may be quite low.


b. Financial consequences of damages caused to car are relatively moderate.
c. Financial consequences of catastrophic loss such as destruction of house are high.
d. All of the above.
e. None of the above.

6. Individual investors who are risk averse would prefer investing in:

38
a. Stock Market
b. Bond Market
c. Derivatives Market.
d. Bank deposits.
e. None of the above.

7. Before deciding upon the specific risk control measures to be adopted it is necessary to
conduct…………………………. Exercise.

a. Risk Analysis.
b. Risk Control
c. Cost Control
d. Cost-Benefit
e. None of the above.

8. The techniques of risk control available for individuals and others are classified under this /
these head / s.

a. Loss Avoidance.
b. Loss Prevention.
c. Loss Reduction.
d. All of the above.
e. None of the above.

9. After receiving a doctor’s advice that there is a risk of premature death by cancer, an
individual might completely stop smoking. It is

a. Loss Prevention.
b. Loss Avoidance
c. Loss Reduction
d. Loss Understanding.
e. None of the above.

10. Most frequently occurring minor health problems (flu, cold etc) which do imply low loss
severity can be reduced in frequency of occurrence by taking appropriate action. It is
a. Loss Prevention.
b. Loss Avoidance
c. Loss Reduction
d. Loss Understanding.
e. None of the above.

11. Individuals investing in equity shares and mutual funds for purposes of wealth
accumulation and retirement needs are invariably subject to investment risk. A risk
reduction technique that is appropriate for mitigating the investment risk is portfolio
diversification. It is

39
a. Loss Prevention.
b. Loss Avoidance
c. Loss Reduction
d. Loss Understanding.
e. None of the above.

12. Individuals might have got numerous risk-financing options which, may however be
classified as-

a. None of these.
b. Risk Retention.
c. Risk Sharing
d. Risk Transfer
e. All of the above.

13. One of the following is not a form of risk transfer;

a. Hold Harmless Agreements


b. Incorporation.
c. Partnership
d. Insurance
e. None of the above.

14. A number of factor /s is / are considered by individuals before making risk-financing


decisions.

a. Expected cost
b. Financial position
c. Degree of risk aversion
d. All of the above
e. None of the above.

15. A house owner includes in the lease agreement a clause making the tenant responsible for
all injuries the guests may suffer while on the leased premises. It is ………. Form of risk
transfer.

a. Hedging
b. Insurance
c. Hold Harmless Agreement
d. Diversification
e. None of the above.
SECTION A – Answers

1.d 2. c 3.c. 4. d. 5. e. 6.d 7. d 8.d. 9.b.10. a. 11. c. 12. e. 13.c. 14.d. 15.c.

40
SECTION B

1. Out line the process of Risk Assessment.

ANS:

The first step in the process of risk management by individuals, businesses and society is the
assessment of risk. Risk assessment comprises identification and analysis of potential loss
exposures. Such assessment has to be continuous and comprehensive. As individuals as well as
their environment are not static but dynamic and do change, risk assessment has to be an ongoing
activity. A person’s risk profile undergoes significant change when he or she marries and has
children. Likewise, a person’s inventory of assets grows with fresh acquisition of assets over time
and, as a result, his property risk profile changes over his life time. The legal environment a person
faces is altered by enactment of new statutes and changes in case law. Because of these and
other changes, individuals will have to continuously and suitably adjust their risk management
efforts.
A business organization’s is exposures to loss is now managed by using a new risk management
technique called risk mapping making use of a matrix plotting in loss frequency and loss severity.
The risk mapping technique might not be particularly useful for managing risk exposures of
individuals and families. However, it might be instructive to understand the technique with a view to
applying it while discussing the techniques of risk control and risk financing for individuals and
families. Loss frequency measures the likelihood of occurrence of a loss event and loss severity
indicates the potential financial implication of the event.
2. What are the techniques of risk control available for individuals and others?
ANS:
The techniques of risk control available for individuals and others are classified under three heads:
a. Loss Avoidance
b. Loss Prevention, and
c. Loss reduction
(i) Loss Avoidance :
By refraining to expose oneself to certain types of losses, one can avoid loss. Transportation risk
can be avoided by not flying, which might not be always possible. Another example of loss
avoidance is discontinuing an activity which an individual might have been engaged in hitherto and
which leads to a loss. For example, after receiving a doctor’s advice that there is a risk of
premature death by cancer, an individual might completely stop smoking. Likewise, discontinuance
of any activity that may contribute to loss. However, one has to note that individuals are not always
rational in making decisions. Furthermore, it may not always be possible to avoid risky activities
fearing potential loss. Moreover, discontinuance of an activity which is involving a loss exposure
might still create a liability loss exposure from the transactions carried out earlier. For example,
withdrawal of a drug by a pharmaceutical company because the firm has come to know of its
dangerous side-effects may still create liability exposure on past scale transactions.

41
(ii) Prevention of Loss:
It is almost impossible to avoid most situations of risk as risk is a part and parcel of everyday life.
As such, individuals and families can take necessary steps to reduce the probability of loss
occurring, where feasible. Action can be taken to ensure that loss frequency is reduced, where
potential frequency of risk is high. For example, most frequently occurring minor health problems
(flu, cold, headache etc.,) which do imply low loss severity can be reduced in frequency of
occurrence by taking appropriate action aimed at loss prevention.
(iii) Loss Reduction :
Loss reduction activities would be appropriate where the potential severity of loss is important. The
activities taken for reduction of risk severity would mitigate the loss, though complete prevention of
loss is not on the cards. The loss reduction activities will hopefully shift the consequences of loss
to the left to the moderate region from the catastrophic region.

3. Explain Risk Retention and Risk Transfer.

ANS:
1. Risk Retention :
Meeting the loss costs by the individual or family by internalizing them is called risk retention. The
losses that are internalized would be financed out of one’s own income or wealth. While the
decision to retain a loss exposure and finance it out of one’s own funds may be a conscious
decision of the individual or family. In many cases, it may be a “default option” or an involuntary
option if the option of loss transfer is not available or is not affordable or if the individual is unaware
of the loss exposure. The expected cost of risk financing through the method of risk retention
involves an uncertain amount for which the individual or family must be prepared.
2. Risk Sharing :
Risk sharing as a technique of risk financing options available to individuals and families is a blend
of the other two financing options, namely risk retention and risk transfer. Most non-life insurance
contracts contain provisions for allocation of losses between insurer and insured. Losses below a
specified size to the borne by the insured, are called deductible provisions. Further, deductibles
operate on a ‘per loss’ basis, that is, the deductible provision is applicable for each loss event.
Deductibles are normally specified in rupee amount. However, in case of coverage of losses
resulting from floods and earthquakes, they are expressed as a percentage of the total value of the
insured’s property. In many health insurance polices, “aggregate” deductibles are specified and the
deducible operative in terms of the cumulative amount. In other words, any loss during a policy
period, in excess of the aggregate deductible amount (which is to be borne by the insured), is the
liability of the insurer.
As regards, provision of co-insurance, the allocation of losses between the insurer and the insured
is expressed on a percentage basis (say 80 percent of the covered losses by the insurer and 20
percent retained by the insured). Co-insurance as a form of loss sharing is quite common in health
insurance. It may also be noted that where health care claim is large, sometimes, health insurance

42
policies contain a provision restricting the loss-sharing upto a specified level of covered loss.
Disability insurers use a waiting period, called an elimination period, as a loss sharing provision.
For example, the insurance policy may stipulate that the initial period of disability, say 15 days, is
retained by the insured.
4. Explain various forms of risk transfer.
ANS:
Risk transfer involves agreement by the transferee to assume the loss or risk that the transferor is
desirous of escaping. The process of risk transfer involves a payment by the transferor to the risk
bearer or transferee. It is normally done through insurance. Through the process of transfer, the
degree of risk is sometimes reduced as the transferee may be in a position to predict loss by
applying the law of large numbers.
Five forms of risk transfer are discussed by insurance text books.
• Hold Harmless Agreements
• Incorporation
• Diversification
• Hedging
• Insurance
5. What is Hold Harmless Agreement?
ANS:
Hold Harmless Agreements : Many kinds of contracts, by incorporating appropriate provisions, may
transfer responsibility for some types of losses to some one else who is not obligated to bear it.
They are also called indemnity agreements. The clauses in the agreements may fix the
responsibility for paying for various losses on some individual or individuals. An example of this
type of agreement : A house-owner including in the lease agreement a clause making the tenant
responsible for all injuries the guests may suffer while on the leased premises. It is a case of
shifting the responsibility for payment of losses to the tenant without any actual reduction in the
original risk because generally no greater predictive ability in regard to potential losses can be
expected on the part of the tenant than on the house-owner.
6. Explain insurance as a risk transfer tool.
ANS:
Insurance As Risk Transfer :
Insurance is considered a special form of the technique of risk-transfer. It is, on one side, an
economic or social institution designed to perform certain special functions and on the other, as a
legal contract between two parties, the insured (transferor) and the insurer (transferee).
Taking the latter aspect first, in certain situations, the best way to manage a particular risk, may be
to purchase insurance. This is because of the insurer’s ability to efficiently handle risk through the
operation of law of large numbers. It does not, however, imply that an automatic assumption that

43
the only way to handle a particular risk exposure is insurance. Such an assumption is not
warranted. Some of the corporate risk managers, in fact, use insurance as a last resort, when other
risk management techniques are not considered adequate by themselves.
Insurance as an economic institution operates on the principle of risk pooling and risk reduction,
besides a mechanism of risk transfer. Pooling is the sharing of total losses among a group. The
aggregate amount of uncertainty is brought down facilitating risk reduction by “combining under
one management a group of objects or persons so that the total losses to which the insured group
is subject becomes predictable within narrow limits”. Through this process, the overall risk for the
group is reduced and the resultant losses are pooled, generally through the method of payment of
an insurance premium. Thus, the insureds, through the mechanism of insurance, transfer specific
risks to the group and exchange a potentially large and uncertain loss for a relatively small certain
payment, i.e. the insurance premium.
7. What factors determine risk financing decisions?
ANS:
A number of factors are considered by individuals before making risk-financing decisions or
choosing the category appropriate to them out of the three available options, risk retention, risk
reduction and risk transfer.
The factors include
a. Expected cost
b. Financial Position
c. Degree of Risk Aversion
d. External Constraints
The expected value and the variability of cost of various risk-financing options are prime
considerations in regard to the choice people make. The financial position of the individual making
the choice also influences his / her decision. The degree of risk aversion of the individual will also
be an influencing factor in the choice of risk-financing decision. Finally, the choice of an individual
of the type of risk financing he or she makes may be constrained by external factors. e.g. Motor
owners are constrained by legislation in many countries to the purchase of motor vehicle
insurance. Another example is the constraint that insurance be purchased for the purpose of
protecting the collateral. When financial institutions extend credit or provide loans for home
purchase or car purchase, they impose a condition that the debtors purchase property insurance.

SECTION C (case studies)

Das and Associates is a management consulting firm with head quarters at Mumbai and operations
spread all over the country. The company was formed in 2000 by its promoter director Anup Das.
Das presently maintains 150 consultants and 100 employees working in supporting roles. The firm
has developed a reputation for being especially helpful in solving personnel and financial
management problems. As Das and his management team plan for the future, they want to be very

44
systematic in identifying key issues, opportunities, and potential problems that may confront the
company in the years ahead.

One area that has not received much attention to date is that of pure risk exposures. Thus, Das
has recently assigned one of his top consultants to take on the responsibility for identifying all of
the potential risks confronting his firm. After the exposures have been identified, Das wants to
know the relative importance of each one. For example, is the risk of loss due to fire potentially
more damaging than the risk of adverse liability judgments ? What losses are most likely to happen
in a given year, and how much would probably be lost in each case. Can consistency be expected
from one year to the next? Das believes that it is impossible to make good risk management
decisions without first having answers to these and similar questions. Discuss.

ANS:

Human nature is fundamentally adolescent (e.g., "bad things won't happen to me") and short-
sighted (e.g., tends toward immediate gratification instead of long-term gratification). Disaster
preparation and most other components of risk management require leadership that is mature
(e.g., "bad things happen all the time and can happen to me as easily as to anyone else) and far-
sighted (e.g., able to delay gratification and make long-term investments for a greater good). To
achieve maturity and far-sightedness, many things are required over many years. Key among
those things are academic education and life experience that include the formation of positive
values, ethics, and knowledge of the political, economic, and business conditions most likely to
produce positive results from human nature.

The first step in the process of risk management by individuals, businesses and society is the
assessment of risk. Risk assessment comprises identification and analysis of potential loss
exposures. Such assessment has to be continuous and comprehensive. As individuals as well as
their environment are not static but dynamic and do change, risk assessment has to be an ongoing
activity. A business organization’s exposures to loss is now managed by using a new risk
management technique called risk mapping making use of a matrix plotting in loss frequency and
loss severity.
The second step in the risk management process is to control the risk by selecting those
techniques that will reduce the frequency of loss and reduce the severity of loss. Before deciding
upon the specific risk control measure to be adopted it is necessary to conduct a cost-benefit
exercise. That is an evaluation of the expected benefits and costs involved to determine whether
the decision taken is economically sound or not.

Despite one’s best efforts, loss exposures can never be eliminated; one has to think of financing
losses, when losses happen. The question of sources of funds for repairing losses is addressed to
techniques of loss financing. Loss financing (and the various sources of funds available for this
purpose) is required to repair or replace damaged property, to meet expenses involved in legal
defence and any advance judgments or award in cases of legal liability, to pay medical bills and
other health care expenses and to replace lost wages and taking care of the financial
consequences of premature death or loss of life.

45
While one end of the risk-financing continuum, is risk retention, the other end of the continuum is
risk transfer. In risk transfer, the financial impact of loss is shifted to another party. Risk transfer
involves agreement by the transferee to assume the loss or risk that the transferor is desirous of
escaping. The process of risk transfer involves a payment by the transferor to the risk bearer or
transferee. It is normally done through insurance.

Basing on the above analysis the consultants need to look at all dimensions and estimate probable
causes of risks, how to manage them, and how to fund them in case really it occurs.

46
CHAPTER 7

RISK MANAGEMENT BY BUSINESSES AND ENTERPRISE RISK MANAGEMENT (ERM)

SECTION - A
1. One of the following is not true;

a. “It is better never to have a suffered a loss than to so suffer and to collect under an
insurance policy”.
b. The enterprise might fail entirely notwithstanding that it has the finest risk-financing
programme it could ever devise.
c. There is ample evidence that a significant proportion of firms never fully recover
from the effects of a major fire and some have to be wound up within a short time
even if fully insured.
d. A reduction in loss expectancies brought about by risk control measures will
usually also reduce the cost of the risk-financing programme.
e. None of the above
2. One of the following is not true;
a. all exposures to risk must be identified;
b. all exposures need to be evaluated according to cause and effect ,the aim being to
quantify probabilities and severities;
c. the possibility of avoiding or eliminating any of the risks should be investigated, and if
feasible the appropriate steps should be taken;
d. in the case of other risks, risk reduction measures need to be explored and
implemented;
e. None of the above
3. Business enterprises normally do not face this risk;
a. Interest Rate Risk
b. Foreign Exchange Risk
c. Commodity Price Risk
d. Personal Accident Risk
e. None of the above.

4. One of the following techniques can be adopted by a business enterprise to transfer risk;
a. Diversification.

47
b. Hedging
c. Incorporation.
d. A&B
e. All the above.

5. A oil company, in order protect itself from future price risk, enters into a futures agreement
with another party. It is case of
a. Speculation
b. Price Protection
c. Hedging
d. Gambling
e. None of the above.

6. A firm’s uninsured losses are ultimately paid by


a. Managing Director
b. Share Holders
c. Debtors of the company
d. Tax payers in the society
e. None of the above.

7. The economic reason / s for the corporate business units engaging in firm-specific risk
management activities
a. Risk transfer confers benefits on the firm managing its risk exposure through risk
transfer by covering potential bankruptcy costs.
b. While shareholders can achieve reduction in firm specific risk through portfolio
diversification, the other stake holders are not in such a comfortable position.
c. Risk transfer that the cash flows will be available to meet the obligations to debt
holders and for future investment projects that enhance the firm’s value.
d. A&B
e. All the above

8. One of the following is not true;

48
a. Credit and monetary risk is handled by the Corporate Treasurer
b. Reputation risk is handled by marketing and public relations personnel.
c. For handling operational and commodity risk, strategic business units develop
appropriate controls.
d. Financial risk is handled by CEO of the company.
e. None of the above.

9. One of the following is not financial risk:


a. Interest Rate Risk
b. Price Risk
c. Credit Risk
d. Foreign Exchange Risk
e. Operational Risk

10. An earth quake damage to a multinational company’s property in a country would less
likely to have any correlation with its exposure to interest rate or foreign exchange risk in a
country located in another part of the world. This is
a. ‘Silo’ approach.
b. ‘Natural hedging’
c. Diversification
d. International Management
e. None of the above.

11. Enterprise Risk Management normally does not cover one of the following activities;
a. Technological changes that affect a firm’s strategic goals
b. Temporary break down of an enterprises EDP system
c. Changes in regulatory regime that adversely affect the business environment.
d. Stock market upheavals affecting the pricing structure of an enterprise
e. None of the above.

49
12. This insurance is based on spreading risk over time.
a. Financial insurance.
b. Infinite risk insurance.
c. Finite risk insurance.
d. Time risk insurance
e. None of the above.
13. It is a process of creation of securities such as bonds, derivative contracts (Futures,
Options, Swaps etc.,) whose price movements are linked to an insurance risk.
a. Securitization.
b. Bonding
c. Factoring
d. Forfeiting
e. None of the above.
14. It is a choice to buy or sell the asset at some future date at an agreed price;
a. Option.
b. Futures.
c. Forwards.
d. Collar.
e. None of the above.

15. A call holder is always speculating on…………….in prices.


a. Rise
b. fall
c. stability
d. fluctuation
e. None of the above.
SECTION A (Answers)
1.e. 2.e 3.d. 4. e. 5. c. 6.d 7.e 8.d. 9.e.10.b. 11. d. 12. c. 13.a. 14.a. 15.a.

50
SECTION B
1. “Regardless of the techniques that may be employed at each stage , every risk
management programme must proceed according to the logical sequence of events if it is
to stand any chance of success:”. Explain
ANS:
• all exposures to risk must be identified;
• all exposures need to be evaluated according to cause and effect ,the aim being to
quantify probabilities and severities;
• the possibility of avoiding or eliminating any of the risks should be investigated,
and if feasible the appropriate steps should be taken;
• in the case of other risks, risk reduction measures need to be explored and
implemented;
• the residual risks need to be evaluated so that decisions can be taken about the
best methods of financing them; and finally
• the results of the whole programme need to be monitored and regularly reviewed
in the light of changing conditions .
2. Explain Diversification and Hedging as tools of handling risks.
ANS:
Diversification policy adopted by a business firm across various lines of businesses and / or
geographic locations, mainly from the angle of business synergies, economies of scale and cost
reduction, also helps the firm transfer risk across business units. Furthermore, combining in one
firm of different lines of business and / or different geographical locations may help in the reduction
of total risk exposure by the firm through what is known as the portfolio effect of pooling individual
risks with different co-variances.
Assume, a business unit has production facilities at two different location. Heavy floods might have
caused damage to the production facilities in Assam; it is highly unlikely that the same floods would
cause any damage to the second production facility located in Andhra Pradesh.
Hedging is a means of transferring a speculative risk (risk of fluctuations in price) to a third party –
either a speculator or another hedger. For example, a oil company, to protect itself against price
risk (fluctuations in prices of jet fuel) enters into a futures agreement with a counter-party, say a
speculator. In effect, the speculator (counter-party) is assuming the price risk associated with jet
fuel transferred by an oil company which produces and supplies jet fuel to the oil market. If by the
hedging agreement, the operating profits of the oil company improve, the project involving
transferring of price risk to a speculator in the futures market yields net present value to the firm
and should increase the firm’s value.
3. Outline some of the risks covered under Enterprise Risk Management.

51
ANS:
1. Damage to a crop affecting the quality, quantity or marketability of grain suppliers.
2. Technological changes that affect a firm’s strategic goals.
3. Temporary breakdown of an enterprises electronic data processing system.
4. Disruptions to the production activities of a firm by political upheaval.
5. Adverse commodity price changes
6. Changes in regulatory regime that adversely affect the business environment.
7. Lapses in due diligence relating to Mergers and Acquisitions.

4. Explain the following:


a. Captive Insurance Companies.
b. Finite Risk Insurance.
c. Multi-year insurance contracts.
d. Multiple Trigger Contracts.
ANS:
Captive insurance companies: A captive insurance company is owned by a non-insurance
business unit for the purpose of accepting the risk exposures of the parent company. It is a captive
of the parent because it is under the control of the latter. The captive may be on shore (one
domiciled in the country) type or off-shore type (in another country in which there are less
restrictive regulatory laws, in regard to minimum capital requirements and investment allocation).
Establishing a captive insurance company, however, may take considerable time and substantial
money may be required.
Finite risk insurance. Unlike traditional insurance contracts which are based on spreading across
a pool of similar risk exposures, finite risk insurance contracts are risk transfer contracts based on
the concept of spreading risk over time. In this type of contracts, the insured transfers limited
extent of risk. The main objective of such contracts is smoothing the losses during the contract
period.
Multi-live/multi-year insurance contracts generally combine a number of risks and the policy
period extends over multiple years. The risks that are combined may be pure risks such as
property, liability and auto risks or combination of pure risks with financial risks.
Multiple – Trigger contracts pool risks which together could seriously affect the value of the firm.
For the shareholders of a company the effect of the risk on the firm’s earnings is important, not the
source of the risk. Multi-trigger policies generally combine a pure risk with a financial risk. The
policy is “triggered” and payment is effected only, when the risky event in each category occurs.
For example, an electric power generation company might buy a policy that is “triggered” if one of
the firm’s power generation plants breaks down (the first risk), during a period when the power
rates are marked by considerable volatility (the second risk).

52
SECTION C (case studies)

The global economy has become a reality for many firms. As international operations expand, it is
important for management to recognize that the same risk may result in very different estimates for
the maximum probable loss and maximum possible loss in different countries. With differing loss
frequency and severity estimates, the most appropriate risk management techniques are also likely
to vary from country to country. Several examples illustrate this point.

In the United States, risk managers often concentrate much time and effort in identifying,
evaluating and managing liability exposures. However, the rest of the world is not as litigious, thus
making liability losses outside of the United States less problematical. The analysis of potential
property losses may also vary. For instance, the proliferation of very old buildings through out
Europe presents unique problems. In Pacific Rim countries, the potential for catastrophic loss due
to typhoons or other windstorms is very real. Risk evaluation in countries experiencing frequent
currency devaluation (such as Brazil) can also be challenging, because property valuations
become obsolete so quickly.

Risks of loss due to crime also vary considerably from one country to the next. For example, in
Eastern Europe and the former Soviet Union, deficiencies in the computer infrastructure makes it
nearly impossible to track stolen vehicles in the same way that it is possible in most Western
countries. And the risk of loss to employees due to kidnapping or other violence is well known in
many Latin American countries. Often, the dynamic risk of political unrest merely exacerbates such
problems, making the risk manager’s job continually challenging.

Discuss the challenges of Risk Managers in the light of changing global scenario and Enterprise
Risk Management practices.

ANS:

The optimal RM Department reporting structure is difficult to define and can vary with the size
and scope of the business operation. While I agree in general, the wrong reporting relationship
can certainly undermine the RM department's effectiveness. In smaller organizations, the RM
may need to report to the president or owner, because as was said, every silo of an organization
is touched by risk management and therefore there exists the potential for special interests and
conflicts by individual departments. Risk management affects the overall business operations,
therefore having it report through HR, treasury, finance, or the general counsel is in most cases a
formula for failure.

Risk manager must, above all, be a tough-minded individual able to communicate because the
task at hand is, in large part, selling the concept of risk management to VP-level peers. The VPs
are paid to grow revenue, the risk manager is paid to prevent/limit loss. There is a healthy tension
here which, in the end, serves the organization best. If the risk manager isn'tgetting results, then
find a risk manager tough enough for the job. Investing more authority in an essentially weak
player will only foster the wrong kind of stress within the organization—and make life even more

53
player will only foster the wrong kind of stress within the organization—and make life even more
difficult for the risk manager.

Business enterprises face a number of types of risks – some called pure risks such as property
and liability risks and some other risks that are called financial risk such as interest rate risk,
foreign exchange risk and commodity price risk. The former that is “pure” risks are often insurable
and are characterized by only a downside potential i.e. chance of loss. Insurable risks can be
managed by insuring them. The compartmentalization of risk management activities within a firm
may be illustrated: pure or hazard risk is managed by the traditional risk manager; credit and
monetary risk is handled by the corporate treasurer; marketing and public relationship personnel
focus on reputational risks; For handling operational and commodity risk, strategic business units
develop appropriate controls. New financial products – futures, options, swaps and other
derivative contracts are used for managing financial risks, such as interest rate risk, commodity
price risk and foreign exchange risk.
During the last decade or so, there is a proliferation of a number of tools of risk transfer, (as an
alternative to insurance). It is true that they have not become widely used. They include creation
of captive insurance companies, finite risk or financial insurance, multi-line/multi-year insurance,
multitrigger policies and securitization techniques.
Among the risk management instruments commonly used by enterprises, hedging tools or
insurance are the most important. These devices permit the party holding some risky asset to
transfer that risk to another, called counter-party. The asset value can be separate from risk and
the latter sold separately for a price. Risk is a traded commodity and is sold in the market for risk.

54
CHAPTER 8

REINSURANCE THEORY

SECTION A

1. The agreement between the primary insurer and reinsurer is known as:

a. Coinsurance
b. Under Insurance
c. Over Insurance
d. Reinsurance
e. None of the above.
2. The other names for primary insurer;

a. Insured
b. Ceder
c. Cedent
d. Proposer
e. None of the above.

3. Reinsurance is a contract of –
a. Indemnity
b. Securitisation
c. Aelatory contract
d. Both A & B
e. Both A & C

4. An insurer’s ability to provide a high limit of insurance on a single loss exposure, is called;

a. Solvency syndrome.
b. Exposure Barrier
c. Large-line Capacity
d. Over Capacity
e. None of the above.

5. Which one the following can be classified as one of the primary functions of reinsurance?

a. Stabilizing of loss experience.


b. Minimizing the effects of catastrophe.
c. Increasing the capacity.
d. All of the above
e. None of the above.
6. The process of reinsuring all losses for an entire class, territory or book of business is
known as
a. Catastrophe Reinsurance
b. Disaster insurance

55
c. Portfolio Reinsurance
d. Large-line capacity
e. None of the above.
7. At law, the policy holders do not have direct access to the reinsurers. However there is an
exception and it is –
a. Provision of Subrogation
b. Cut-through Endorsement
c. Pre-emptive Clause
d. All of the above
e. None of the above.
8. Reinsurance contract under which the ceding company has the option to cede and
reinsurer has the option to accept a risk of a specific business line is called…….

a. Facultative Reinsurance
b. Treaty Reinsurance
c. Proporional Reinsurance
d. Optional Reinsurance
e. None of the above.
9. What does the word “ treaty” mean in Treaty Reinsurance
a. An agreement between two parties
b. Bulk Insurance policy
c. Single Insurance policy
d. Ceding
e. None of the above
10. Under this the primary insurer cedes a fixed, predetermined percentage of every risk to the
reinsurer within the class or classes subject to the treaty.

a. Surplus Share Treaty


b. Excess of Loss Treaty
c. Quota Share Treaty
d. Stop Loss Treaty
e. None of the above.
11. In surplus reinsurance, the retained business is called

a. Margin
b. Line
c. Treaty
d. Loss Ratio
e. None of the above

12. The form of excess of loss reinsurance which indemnifies the reinsured against the
amount by which the reinsured losses incurred during a specific period exceed either an
agreed amount or agreed percentage is called

a. Stop-Loss Reinsurance
b. Excess of Loss reinsurance
c. Quota Share Reinsurance

56
d. Proportional Reinsurance
e. None of the above.
13. This is the similarity between insurance and reinsurance

a. Liability of reinsurers is several and not joint


b. Principle of utmost good faith
c. Principle of Indemnity
d. Both B & C
e. Both A & B
14. The sum of all individual losses directly occasioned by any one disaster, accident or loss
or series of disasters arising out of one event, which occurs within one State.

a. Catastrophe Loss
b. Disaster syndrome
c. Loss Occurrence
d. Hurricane clause
e. None of the above.

15. The primary insurer sends a report of reinsured business every month / quarter known as
‘bordereau’ to reinsurer. Such ‘bordereau’ contains

a. Premium details.
b. Details of policies
c. Details of claims
d. A&B
e. A,B & C.
SECTION A (Answers)

1.d 2. c 3.e. 4.c. 5.d. 6.c 7.b 8.a. 9.a.10.c. 11.b. 12.a. 13.d. 14.c. 15.d.

SECTION B

1. Do you agree that reinsurance contracts are aelatory in nature? If so, explain how.

Ans: All the legal principles that govern an insurance contract will be applicable to a
reinsurance contract as well like insurer and reinsurer should have the legal capacity to
contracts and there must be consideration for the contract. Reinsurance contracts are aelatory
in the sense that premium share to the reinsurer will be small compared to the reinsurance
payments that will be made by the reinsurer in the event of a claim.

2. What are the functions of reinsurance?

Ans: there are six different functions of reinsurance that have been broadly recognized:
Stabilisation of loss experience.
Large-line capacity
Financing (Surplus relief)

57
Catastrophe Protection
Underwriting Assistance
Withdrawal from a territory or class of business.

3. Explain Facultative Reinsurance and Treaty Reinsurance.

Ans: Facultative is optional – not compulsory. The primary insurer need not insure every
business. He need only reinsure when there is lack of capacity regarding a particular class of
insurance e.g aviation insurance. Facultative reinsurance is defined as reinsurance contract
under which ceding company has the option to cede and reinsurer has the option to accept a
risk of a specific business line.

Facultative can be either proportional or non-proportional. It plays a very important role in


Aviation Hull, Oil and Space industry.

Where as under Treaty Reinsurance the primary insurer is obliged to cede and the reinsurer is
obliged to accept part or all of the classes of business covered by the treaty. The treaty can be
either proportional or non-proportional basis. The treaty does not require the ceding company
to provide the details of the risks individually but some times a bordereaux is provided.

4. What is proportional and non-proportional reinsurance?

Ans: In proportional reinsurance, the cession accepted by the reinsurer has a certain fixed or
agreed proportion to the retention of the primary insurer. Similarly, when claim occurs, the loss
is settled between the primary insurer and the reinsurer in the same proportion as their
acceptance of sum insured. Proportional contracts can be facultative, quota-share treaty,
surplus treaty, facultative obligatory treaty, open cover and pools. It is also known as
Participating Reinsurance or Pro rata reinsurance.

Under non-proportional reinsurance, the distribution of liability between the cedent and the
reinsurer is based on loss and not on the amount insured. There is no proportionate sharing of
loss but an amount is decided and any loss beyond that amount is paid by the reinsurer. Some
of the non-proportionate contracts are excess of loss treaty and stop-loss treaty.

5. Explain the Excess of Loss Reinsurance.

Ans: In this type of reinsurance, no insurance amount is ceded under excess of loss treaties,
but only losses and premiums. The reinsurance premium is negotiated by line and by insurer.
There are no ceding commissions under excess of loss treaty. The excess reinsurer is only
responsible for losses that exceed the retention and fall within the coverage provided by the
reinsurance contract.

The excess of loss covers are two types: a) per risk cover b) Per event covers / Catastrophe
covers.

The purpose of per risk cover is to reduce or replace the normal proportional reinsurance.
These treaties are referred to as underwriting or working covers. The reinsurer has to pay any

58
loss on an individual risk in excess of the ceding company’s retention. The reinsurer is liable
for all or part of loss to any one exposure in excess of the retention and up to the agreed
reinsurance limit.

Catastrophe covers / Per Event covers: this cover indemnifies the reinsured company for the
amount of loss in excess of a specified retention with respect to accumulation of losses
resulting from a catastrophic event or a series of events.

SECTION C ( Case studies)

1. New Star insurance company has purchased from MumRe Reinsurance company a Quota
Share Treaty with a Rs 250,000 limit and a retention of 25 percent and a cession of 75
percent. New Star has written three policies. Policy A insures building A for Rs 10,000 for a
premium of Rs 100, with one loss of Rs 8000.

Policy B insures building B for Rs 100,000 for a premium of RS 1000, with one loss of Rs
10,000. Policy C insures building C for Rs 150,000 for a premium of Rs 1500, with one
loss of Rs 60,000.

Rahul khanna, the new Reinsurance Manager, feels that Surplus Share Treaty is better
than Quota Share Treaty as it will be limiting the claim exposure of New Star Insurance
and there is more liability in the court of MumRe. Assuming that New Star has purchased a
Surplus Share Treaty with a retention of Rs 25,000 and a limit of Rs 250,000 ( a ten-line
surplus treaty), show your calculations under two reinsurance arrangements.

SOLUTION:

QUOTA SHARE TREATY


Division of Insurance, Premium and Losses

New Star MumRe Total


Rs Insurance (25%) (75%)

Policy A

Insurance 2500 7500 10,000


Premium 25 75 100
Loss 2000 6000 8,000

Policy B

Insurance 25,000 75,000 100,000


Premium 250 750 1000
Loss 2500 7500 10,000
Policy C

59
Insurance 37,500 112,500 150,000
Premium 375 1125 1500
Loss 15000 45000 60,000

SURPLUS SHARE TREATY


Division of Insurance, Premium and Losses
Retention Rs 25,000 and Limit Rs 250,000

New Star MumRe Total


Rs Insurance (%) (%)
Policy A

Insurance 10,000 (100%) 0 (0%) 10,000


Premium 100 0 100
Loss 8000 0 8,000

Policy B

Insurance 25,000 (25%) 75,000 (75%) 100,000


Premium 250 750 1000
Loss 2500 7500 10,000

Policy C
Insurance 25,000(16.67%) 125,000 (83.33%) 150,000
Premium 250 1250 1500
Loss 10,000 50,000 60,000
Thus under Surplus Share Treaty New Star insurance company can limit the payment of
premium and also reduce loss exposure.

60
CHAPTER 9
METHODS OF REINSURANCE
SECTION A

1. One of the following is not a benefit of Treaty Reinsurance;

a. No need to offer each and every risk to Reinsurer.


b. Automatic protection for insurer.
c. Administrative convenience.
d. Premium concession
e. None of the above.

2. The kind of treaty where the cedent company retains the risks till a certain limit
and reinsures only the part that is above its net retention is
a. Excess of Loss Treaty
b. Stop Loss Treaty
c. Quota Share Treaty
d. Surplus Treaty
e. None of the above.

3. It is an agreement under which the reinsured and the reinsurer are obliged to cede
and accept a certain percentage of the risks.
a. Excess of Loss Treaty
b. Stop Loss Treaty
c. Quota Share Treaty
d. Surplus Treaty
e. None of the above.

4. Under this agreement the reinsured limits the amount that it is ready to lose. The
reinsured relieves the excess amount which is in excess of its loss retention.
a. Excess of Loss Treaty
b. Stop Loss Treaty
c. Quota Share Treaty
d. Surplus Treaty
e. None of the above.

61
5. Under this agreement the reinsurer indemnifies the reinsured against the amount
by which the reinsured loss incurred during a particular period exceeds either an
agreed amount or an agreed percentage or some other percentage.

a. Excess of Loss Treaty


b. Stop Loss Reinsurance
c. Quota Share Treaty
d. Surplus Treaty
e. None of the above.

6. The reinsurance in which the reinsurance limit and the reinsurer’s loss limit apply
‘per risk’ is called

a. Per occurrence reinsurance


b. Per risk reinsurance
c. Catastrophe reinsurance
d. Aggregate reinsurance
e. None of the above.

7. One of the following is not a characteristic of Quota Share Treaty

a. Sharing of percentage is fixed.


b. No individual cessions
c. Premiums and losses are settled by account.
d. Individual cessions
e. None of the above.

8. One of the following is not a characteristic of Pro-rata reinsurance.

a. Liability is based on predetermined percentage


b. It is proportional treaty
c. There is sharing of risks.
d. It is non-proportional treaty
e. None of the above.

9. It offers the insurer protection against the accumulations resulting from numerous
losses caused by the same event (cyclone, earthquake)

a. Pro rate reinsurance


b. Peril reinsurance
c. Cover per event
d. Both A&B
e. None of the above.

62
10. a ‘line’ is equal to the ceding company’s retention. Where, a ceding insurer has a
ten line surplus treaty on the basis of a maximum retention of Rs 5000, the
capacity of the treaty to absorb liability over and above retention would be
a. Rs 50,000
b. Rs 55,000
c. Rs 5,000
d. Rs 1,00,000
e. None of the above.

SECTION A (Answers)

1.d 2. d 3.c. 4.a. 5.b. 6.b 7. d 8.d. 9.c.10. a.

SECTION B

1.Explain Quota Share Treaty.

Ans:

In Quota share contracts the cedant binds itself to retain and cede fixed proportions of all the
business it underwrites up to a fixed amount.
For example, if the Ceding Company shall retain for own account 40% of all burglary business, with
an underwriting limit of 50,000 per risk. The Cedant shall reinsure with the Reinsurer, who agrees
to accept a 60% share of all burglary business.
The advantages of the system is particularly appropriate in the following cases:
1. When a company commences business in a line of business for which no statistics
exist; here the Reinsurer participates in the underwriting of each policy, large and
small, and pays in the same proportion its share of the losses.
2. In order to simplify administrative work and reduce cost.
3. If the loss ratio has got out of the Cedant’s control and cannot be corrected
immediately without endangering the relationship with the clientele. Under these
circumstances Cedant would conclude quota share reinsurance as a relief for a limited
period of time.

2.Differentiate between Proportional and Non proportional treaties.

ANS:

Non-proportional reinsurance arrangements are characterized by a distribution of liability between


the Cedant and the reinsurer on the basis of losses rather than Sums Insured, as in case of
proportional arrangements.

63
As compensation for the cover granted, the Reinsurer receives part of the original premiums and
not the part of the premium corresponding to the sum reinsured as in proportional reinsurance.
The following common characteristics differentiate them from proportional treaties.
1. The size of cession is not determined case by case.
2. Administrative costs are substantially reduced.
3. Usually there is no profit commission.
4. Reinsurance premium is worked out on the basis of exposure and past loss experience.
3.Differentiate between Surplus Share Treaty and Quota Share Treaty

ANS:

Surplus share treaty Quota share treaty


1. Often property only. 1. Both property and casualty.
2. Sharing of percentage is variable. 2. Sharing of percentage is fixed.
3. Individual cessions. 3. No individual cessions.
4. Reinsurer’s premium is the percentage 4. Reinsurer’s premium is the
of the original premium less a percentage of the original
negotiated ceding commission.
premium less a negotiated
ceding commission.
5. Premiums and losses are settled by 5. Premiums and losses are settled
account. by account.

4.Differentiate between Pro-rata and Excess of Loss Reinsurance.


Ans:
Pro-rata Excess of loss

1. Liability is based on predetermined 1. Liability in excess of the


percentage Cedant’s retention.
2. It is a proportional treaty. 2. It is a non-proportional treaty.
3. There is sharing of risks. 3. Risks that are above retention
are not shared.
4. This treaty focuses on the size of 4. This treaty focuses on the size
the risk. of the loss.
5. Rate is calculated as the percent of 5. There is a separate rate, with the
original premium less the ceding commission or without
commission. commission.

64
6. Premiums and losses are settled by 6.Settlement of premiums
account or bordereau. account and the settlement of
losses individually.

5. Outline the characteristics of Non-proportional treaties.

ANS:
Per risk Per Occurrence Aggregate (stop loss)
1. It is negotiated on rate 1.It is negotiated on rate 1.It is negotiated on Rate
exposure basis and there exposure basis and there exposure basis and
is no commission. is no commission. there no commission.
2.Premiums are settled by 2.Premiums are settled by 2.Premiums are settled by
annual adjustment of annual adjustment of annual adjustment of
deposit premium. deposit premium. deposit premium.
3. Premium is minimum. 3. Premium is minimum. 3. Premium is minimum.
4. Losses are settled 4. Losses settled by 4. Losses are settled
individually. catastrophe or event. annually
5. Retention is for each risk 5. Retention is usually 5. Retention and thelimit
or building or location. above a minimum of two stated as a loss ratio.
Full-losses.
6. It has per occurrence 6. It has a co-insurance 6. It has a co-insurance
limitation. provision when the provision when the
reinsured shares the loss reinsured shares the
above retention. Loss above retention.

SECTION C (case studies)

1. Munich insurance company entered into two surplus treaty contracts with the reinsurers. The
first surplus treaty consisted 10 lines with a maximum liability of Rs. 30,00,000.The second
surplus treaty consists of 20 lines with a maximum liability of 50,00,000.The claims filed are as
under: Show calculations of premium as per surplus treaty.
Risk Gross sum insured Retention applicable
1 Rs. 2,00,000 Rs.2,00,000
2 Rs. 3,00,000 Rs.1,00,000
3 Rs. 12,00,000 Rs.2,00,000
4 Rs. 30,00,000 Rs.4,00,000
5 Rs. 65,00,000 Rs.2,00,000

SOLUTION:

65
Risk Gross sum Retention [Rs] Cessions to 1st Cessions to 2nd
insured [Rs] surplus treaty surplus treaty
1 Rs. 2,00,000 Rs.2,00,000 Nil
2 Rs.3,00,000 Rs.1,00,000 Rs.2,00,000 Nil
3 Rs.12,00,000 Rs.2,00,000 Rs.10,00,000 Nil
4 Rs.30,00,000 Rs.4,00,000 Rs.26,00,000 Nil
5 Rs.65,00,000 Rs.2,00,000 Rs.20,00,000 Rs.40,00,000
balance 3,00,000 has to be arranged facultatively.

2. Rakshak Insurance was prepared to bear any claim up to Rs 3,00,000. It estimated the
maximum exposure per event to be Rs 12,00,000. Rakshak Insurance entered an excess
of loss reinsurance contract with Takat Reinsurance to meet the balance of any claim up to
further Rs 9,00,000 in excess of Rs 3,00,000 per risk.
On account of a cyclone the town suffered heavy damages and several
houses were badly hit. 60 houses of the insured were damaged.
Subsequently 60 claims of each Rs 3,60,000 were filed with Rakshak.

Show the calculations relating to – a) retention amount by the insured


b) recoverable amount from the excess of loss reinsurance.

SOLUTION:

Number of Houses 1 (Rs) 60 (Rs)

Payment to primary insured 3,60,000 2,16,00,000

Retained by Rakshak 3,00,000 1,80,00,000

Recovery from Takat Re 60,000 36,00,000

66
CHAPTER 10

TREATY WORDINGS

SECTION – A

1.One of the following is not true;

a. There must be identity of views between the contracting parties in case of reinsurance
contracts.

b. The parties to the contract should not have any legal disabilities.

c. There must be consideration for the contract.

d. Wordings of the reinsurance agreement are agreed between the cedent and the reinsurer
before hand.

e. None of the above

2. One of the following is not true;

a. Other than facultative reinsurance, all other agreements between the ceding primary
insurer and the reinsurer are written.

b. Treaty wordings have come to occupy an important place in the study of reinsurance.

c. Treaty wordings are standardized throughout the world.

d. They include agreements on the follow up of accounting procedures.

e. None of the above.

3. “ The liability of the Reinsurer in respect of reinsurance allotted hereunder shall commence
simultaneously with that of the company as soon as the retention of the company on any one
risk as defined by its limits, records, practice or instructions is exceeded”. This clause is known
as:

a. Operative clause

b. Attachment of Cessions clause

c. Exclusions clause

d. Reserves clause

e. None of the above.

4. “ All acceptances hereunder shall be at the same gross rates, terms and conditions as and
to follow the settlement of the company, and the Reinsurer shall follow the fortunes of the

67
company in regard to the cessions in which the reinsurer by virtue of this agreement takes
part”

a. Operative clause

b. Follow the Fortunes clause

c. Exclusions clause

d. Reserves clause

e. None of the above.

5. “ In no event shall this agreement protect the company in respect of : war and civil war
obligatory reinsurances. Any loss or liability accruing to the company, directly or indirectly
and whether as insurer or reinsurer, from any pool of insurers or reinsurers formed for the
purpose of covering atomic or nuclear energy risks”

a. Operative clause

b. Attachment of Cessions clause

c. Exclusions clause

d. Reserves clause

e. None of the above.

6. Which of the following clauses ensures that the reinsured does not change its underwriting
practices with regard to the business covered by the treaty?

a. Change of law clause

b. Costs clause

c. Currency clause

d. Change of underwriting policy clause

e. None of the above.

7. Which of the following clauses allows the reinsured to effect other reinsurances in priority?

a. Recital clause.

b. Records clause.

c. Operative clause.

d. Net retained lines clause.

e. None of the above.

68
8. The reinsurers hereby agree to indemnify the company for that part of the ultimate net loss
which exceeds the amount stated in item… of the schedule on account of each and every
loss occurrence and the sum recoverable under this agreement shall be up to but not
exceeding the amount stated in item… of the schedule, ultimate net loss on account of
each and every loss occurrence’

a. Operative clause

b. Insuring clause

c. Exclusions clause

d. Reserves clause

e. None of the above.

9. “If during the currency of this agreement any balances under any other treaty or treaties
between the company and the reinsurer remain unpaid by one party, the other shall be
entitled to: retain the balance due hereunder until full payment has been made under the
other treaty or treaties: or adjust such balance against the amount due from the other party

a. Reserves Clause

b. Adjustment Clause

c. Set-off Clause

d. Alterations Clause

e. None of the above.

10. Arbitration clause for disputes provides for appointment of – arbitrator/s.

a.1

b.2

c.3

d.4

e.5

SECTION A (Answers)

1.e. 2.c 3.b. 4.b. 5. c. 6.d 7d. 8.b. 9.c.10.b.

SECTION B

1.Under what circumstances will the reinsurance agreement get terminated?

69
Either party shall be at liberty to terminate it as at 31st December in any year by giving not
less than 90 days ‘ previous notice in writing. Unless the parties otherwise agree, the
Reinsurers will remain liable for all reinsurances ceded under this agreement until their
natural expiry.
In the event of War (whether declared or not) arising between India and the country in
which the Reinsurers reside or carry on business or are incorporated, this Agreement shall
be automatically terminated forthwith.
If during the period of this Agreement, postal and/or telegraphic communications should be
rendered impossible as a consequence of War, warlike operations, blockade, revolution,
civil war or other similar event for a period exceeding thirty (30) days, the Company shall
be entitled to terminate this Agreement forthwith without giving notice.
Either party shall have the right to terminate this Agreement immediately by giving the
other party notice:
If the performance of the whole or any part of this Agreement be prohibited or rendered
impossible de jure or de facto in particular and without prejudice to the generality of the
preceding words in consequence of any law or regulation which is or shall be in force in
any country or territory or if any law or regulation shall prevent directly or indirectly the
remittance of any or all or any part of the balance or payments due to or from either party.
If the other party has become insolvent or unable to pay its debts or has lost the whole or
any part of its paid up capital or has had any authority to transact any class of insurance
withdrawn or canceled or suspended or made conditional.
If there is any material change in the ownership or control of the other party.
If the country or the territory in which the other party resides has its head office or is
incorporated shall be involved in armed hostilities with any other country whether war be
declared or not or is partly or wholly occupied by another power.
If the other party shall have failed to comply with any of the terms and conditions of this
Agreement.
2.Explain the Accounting of Losses:

Ans: Preliminary loss advices shall be sent by the Company to the Reinsurer in respect of all
losses where the proportion of the reinsurers sharing in the Agreement is estimated to exceed the
amount stated in the Schedule. Furthermore, the Company shall furnish the Reinsurer an estimate
of outstanding losses as at 31st December of each year.
When the proportion of a loss and or expenses falling upon all the reinsurers sharing in this
Agreement amounts to or exceeds the amount specified in the Schedule, the reinsurers shall be
liable to pay its proportion within 21 days of demand. The Company shall debit all other losses
and/or loss expenses in the accounts of the quarters in which the losses and/or loss expenses are
settled.
Any loss or claim or compromise thereof and all expenses including fire extinguishing expenses
shall be settled by the Company without reference to the reinsurer and such settlements including
ex gratia payments shall in all cases be unconditionally binding upon the reinsurer. The Company

70
at its sole discretion may commence, continue, defend, compromise, settle or withdraw from any
actions, suits and prosecutions and generally do all such matters relating to any loss or claim which
in its judgment may be advantageous and the payment of all expenses and allowances in
connection therewith shall be shared by the reinsurer in proportion to its participation.
The Reinsurer shall share in proportion to its participation in all amounts which may be recovered
by the Company in respect of any loss or claim.”

3. What do you understand by Insuring clause?

Ans: “The Reinsurers hereby agree to indemnify the Company for that part of the ultimate net loss
which exceeds the amount stated in item … of the Schedule on account of each and every loss
occurrence and the sum recoverable under this Agreement shall be up to but not exceeding the
amount stated in item … of the Schedule, ultimate net loss on account of each and every loss
occurrence.
The underlying loss stated in item … of the Schedule shall be retained net by the Reinsured
subject only to underlying excess catastrophe reinsurances as specified in item … of the
Schedule.”
4.Explain the importance of Treaty wordings.

Ans: Written agreements are more easily enforceable than verbal ones and so every insurance
contract is written, satisfying all the legal principles on which insurance is founded and the points of
law to satisfy the definition of a legal contract.

Reinsurance is similar to an insurance contract and hence it is important that the wordings of the
reinsurance agreement are agreed between the cedant and the reinsurer before hand.
Other than facultative reinsurances, all other agreements between the ceding primary insurer and
the reinsurer are by means of written and agreed treaties and therefore, the treaty wordings have
come to occupy an important place in the study of reinsurance. However, it is recognized that the
treaty wordings are not standardized and the different countries and reinsurers have adopted their
own typical wordings for treaties. At the same time, the general conditions are fairly uniform for all
treaties but the special conditions are distinct exercises depending on particular needs of the
contracting parties and also the individual classes of insurance. Again, the proportional and non-
proportional treaties have separate distinct wordings, which have some special features
characteristic of the type of treaty. One other interesting feature of all treaty wordings is that they
include agreements on the follow-up accounting procedures, which actually make implementation
of the treaty much easier.
5.What is dispute resolution mechanism available for company and reinsurer?

Ans: “Incase of any disputes between the company and the reinsurers regarding the interpretation
of the agreement or the rights with respect to any transaction involved either before or after the
termination, disputes as such shall be dealt with the single arbitrator appointed in writing by both
the parties. If incase of any failure to agree upon the single arbitrator it can be referred to two
arbitrators of which one is appointed in writing by each of the parties. Incase of any disagreement
between the two arbitrators an umpire is appointed by the arbitrators.

71
These arbitrators or the umpire are required to be appointed in 30 days after such a requisition of
arbitrators made by the parties. These are appointed in writing by the chairman of the Bombay
Regional Committee of the tariff advisory committee. Such appointed arbitrators or umpires are
required to interpret their agreement as a honorable engagement and make their award and serve
the purpose.
The decision of these arbitrators or the umpires is the final and binding as the case may be
inclusive of allocation of costs on both the parties.

SECTION C (case studies)

Dunhill, a subsidiary of a bank, employed a consultancy firm to advise investors about switching
from corporate to pension plans. Dunhill was insured under a banker’s composite insurance policy
issued by the defendant insurers. Under this policy, the insurers agreed to indemnify the company
in respect of its legal liability from any third party claim. Over a period of years, there were
substantial claims by investors who had changed their pensions and who had suffered losses as a
result. It was alleged that the consultancy firm had provided incorrect advice. For the purpose of a
claim on insurers, the company submitted that the claims by investors were a series of claims that
resulted from a single act or omission, namely, a failure on the part of their own management to
train properly the consultancy firm so as to be able to give investors best advice. Insurers argued
that it was the failure on the part of the consultancy firm in each case to give proper advice to the
investor, rather than the failure on the part of management to provide proper training to the
consultancy firm. Discuss.

SOLUTION:

The aggregation clause requires one to ask a matter of common sense whether the series of
claims and questions was the result of an act or omission of that kind. In common sense terms, all
of the third party claims resulted from a failure on the part of management to provide the training
required to enable the consultancy firm to give proper advice to investors. The proper course would
be to identify the dominant, efficient, or real cause of the loss. In the absence of agreement to the
contrary, the presumption would be that the policy was intended to provide cover where an insured
peril was the dominant, efficient, or real cause of the loss, but not otherwise.

Here, the policy provided cover against losses caused by a range of acts and omissions committed
by officers and employees of the insured. It was not difficult to envisage circumstances in which a
single act or omission on the part of one officer or employee might give rise to claims of a sufficient
size to exceed the deductible when aggregated. Whether that failure should be regarded as a
single act or omission on the part of the insured or as a related series of acts or omissions does not
matter. In either case, the insured is entitled to the claims.

2.Carona Insurance entered into a reinsurance contract with Lawson reinsurance company. The
definition of loss in the treaties provide that a “disaster and / or casualty” included all loss ‘resulting
from a series of accidents, occurrences and / or causative incidents having a common origin and
/or being traceable to the same act….. shall be considered as having resulted from a single
accident..’. Carona insurance settled claims, which covered multiple sites in multiple geographic
jurisdictions, by allocating the sites as separate occurrences under its underlying insurance

72
policies. Later Carona sought recovery from Lawson. Carona treated each settlement for each
insured as a single ‘disaster and/or casualty’ under the terms of the applicable excess of loss
reinsurance treaties.

Lawson Reinsurance did not accept Carona’s allocation. Was Lawson right in rejecting Carona’s
allocation?

SOLUTION:

Carona’s allocation of settlements did not fall within the ‘disaster and/or casualty’ language of the
reinsurance treaties.

Carona’s interpretation of the ‘common origin’ language in the treaties was not valid as the words
‘series of ‘ modify ‘common origin’. ‘Aggregation’ was proper only if the occurrences had a common
origin. Here, the claims could not be aggregated since they did not have a common origin. Thus,
Lawson Reinsurance was right in rejecting Carona’s allocation.

73
CHAPTER 11

REINSURANCE ACCOUNTS AND ADMINISTRATION


SECTION – A

1.One of the following is not true.

a. For the success of reinsurance both the primary insurer and the reinsurer must make joint
efforts.
b. They both have duties as well as rights under the treaties.
c. The primary insurer must conduct his underwriting operations satisfactorily.
d. Both A&B.
e. A,B & C.*

2.In case of large losses investigations are conducted by;

a. Primary Insurer
b. Reinsurer
c. Police
d. A&B*
e. None of the above.

3.“ Follow the Fortunes” means:

a. Take as much reinsurance as possible


b. The reinsurer is normally bound by the primary insurer’s actions in the underwriting and
claims matters.*
c. The reinsurer need to follow the reinsurance rules and not primary insurer’s actions.
d. A&B
e. None of the above.

4.The reinsurer will be able to control losses only under following circumstances:
a) Primary insurer has professional staff.
b) There is well designed information system with primary insurer*
c) There is good communication system.
d) A&B
e) None of the above.

5.One of the following is not true:

a. Direct premium data to calculate the reinsurance premium payable to the


reinsurer.
b. Data for individual losses needed to apply treaty limits and excess retentions;
c. Codes for identifying occurrences under casualty ‘clash’ coverage;
d. Information on risks included in the treaty but not ceded for preserving profitability
of the treaty.
e. None of the above.*

74
6. Now a days the primary insurers are sending information through this statement to reinsurers
instead of bordereaux.

a. Statement of Statistics
b. Current Account Statement
c. Ceding statement
d. Scrip
e. None of the above.

7. Disputes between primary insurer and reinsurer are normally settled through

a. Direct negotiations
b. Arbitration
c. Courts
d. All of the above*
e. None of the above

8. One of the following is not true:

a. Reinsurers have very little to do except collecting reinsurance premium;*


b. Some reinsurers minimize their work by preferring large retentions with the
possibility of no claims being presented at all.
c. Reinsurers may be engaged in auditing the underwriting and claims practices of
the primary insurers.
d. A&B
e. None of the above.

9. What is the purpose of collecting and analyzing a large amount of data by reinsurers?

a. Obtain better terms


b. Balance cessions.
c. Solidify relationship.
d. A&B
e. All of the above*

10. “Probable Maximum Loss” is an assessment by

a. The cedent
b. The reinsurer
c. The surveyor*
d. The broker
e. None of the above.
SECTION A (Answers)

1.e. 2.d 3.b. 4.b. 5.e. 6.b 7.d 8.a. 9.e.10.c

75
SECTION B

1.Outline the role of Primary Insurer in reinsurance administration.

ANS:

The primary insurer must conduct his underwriting operations satisfactorily within the guidelines
and expectations of the treaty so that the reinsurer has no surprises coming in the form of large
losses. More, the primary insurer must notify promptly all large losses and the reinsurer must be
given the opportunity to participate in investigation of such losses.
The primary insurer has the freedom to underwrite individual risks and adjust individual claims
once clear-cut underwriting policies are contemplated under a treaty. We conventionally use a
phrase “FOLLOW THE FORTUNES”. It means the reinsurer is normally bound by the primary
insurer’s actions in the underwriting and claims matters.
The primary insurer must have a good or well designed information system to be able to furnish all
necessary information in time for the reinsurer to be able to control losses wherever possible and
to discharge their obligations under the treaty professionally.

2.The more important data that should be capable of being made available from a good information
system used by the primary insurer should include certain elements. Highlight them.

ANS:

a. Direct premium data to calculate the reinsurance premium payable to the reinsurers;
b. Data for individual losses needed to apply treaty limits and excess retentions;
c. The above data for accounting purposes;
d. Codes generated for catastrophe losses;
e. Codes for identifying occurrences under casualty ‘clash’ coverage;
f. Data to determine the portion of policy/ies ceded to each surplus share reinsurer;
g. Separate data on retention, limits, rates, and the reinsurer involved for each facultative
placement;
h. Information on risks included in the treaty but not ceded for preserving profitability of the
treaty;
i. Data on risks excluded under the treaty but underwritten by the primary insurer so as not
to include the same in the reinsurance bordereau;
An accurate and efficient information system helps the credibility of the primary insurer and helps
the renewal of treaties. The primary insurer must make available his books of account for
inspection by the reinsurer.

76
3.Outline the role of reinsurer in the context of reinsurance administration.

Ans: One would think that the reinsurers have very little to do except collecting reinsurance
premium, and paying claims and brokerage commission to brokers and ceding commission to the
primary insurer. Maybe this is so when the treaty relationship is smooth. In fact, some reinsurers
minimize their work by preferring large retentions with the possibility of no claims being presented
at all.
On the contrary, the reinsurers may be engaged in auditing the underwriting and claims practices
of the primary insurer so as to ensure that these are done satisfactorily and as expected. Again,
whenever the losses are large, the reinsurers may like to participate in the investigation of the
same, both to see that proper procedures are in place and to find out the underwriting implications.
Many a time, the primary insurers, both on underwriting and claims issues, openly consult the
reinsurers.
The reinsurers, not only help in stabilizing loss exposures but also positively assist the primary
underwriters in underwriting on account of their superior experience and expertise.
4. Explain how claims are settled under different methods of reinsurance.
ANS:
The procedure differs from treaty to treaty based on individual agreements.
If it is a pro rata treaty, the primary insurer sends a monthly bordereau to the reinsurer, detailing
the premiums due to the reinsurer and claims due from the reinsurer. The primary insurer will remit
the difference to the reinsurer when the premiums exceed the losses and if the losses exceed the
premiums, the reinsurer remits the difference to the primary insurer. If at any time, there are some
exceptionally large losses, then it is the convention for the reinsurer to remit the losses to the
primary insurer before the end of the reporting period.
In the case of excess treaties, as soon as losses exceed the retention, intimation is given and the
reinsurer pays on being given proof of settlement, which is simply a statement of losses paid by the
primary insurer, together with estimates of current reserves.
In the case of aggregate excess treaties, the reinsurers are known to make initial payments say
sixty days after the end of the accounting year. If it is clear that the losses will exceed the retention,
then payments may be made before the end of the year.
5. Explain Closing of Accounts in case of a reinsurance company.
ANS:
Every insurance company while closing its annual accounts at the end of the year, examines all
claims outstanding with respect to inward treaties in the books in order to make a reasonable
estimate for provisions to be made in the revenue accounts, so that it can present as accurate a
picture as possible. Based on the estimates made for gross claims, the company will work out the
outstanding claims position in respect of its various outgoing treaties, which will be included in the
annual accounts as well as advise to the respective reinsurers.

77
Profit and loss account as well as Balance Sheet is as per Insurance Act and IRDA regulations.

SECTION C (Case Studies)


1.Utkal Insurance company has reinsurance arrangements with Great Eastern reinsurance
company, Kolkatta. The premiums to be paid by cedent to reinsurer is based on the rate specified
in the contract and the rate will be applied to the ceding insurer’s total net premium after ceding to
proportional reinsurances.

MARKET FIRE POOL Rs.


Profit as at 31/03/2004 207,161,000
Premium 767,898,000
PC Terms 15% PC on Profit
Up to 10% of Premium
& 75% of balance
Calculate Profit Commission

SOLUTION:

15% PC up to 10% on Premium


10% on Premium 76,789,800
@15% on above 11,518,470(A)
75% of Balance
Profit 207,161,000
Less 10% Premium 76,789,800
Balance 130,371,200
@75% on Balance 97,778,400(B)
Commission (A+B) 109,296,870

2. The Janapriya Insurance company issued its policy in the name of a consumer group and
issued certificates of insurance covering group members. The Surya Re -reinsurer and its
wholly owned subsidiary, the Janapriya, entered into a reinsurance contract where the
reinsurer reinsured 100% of the janapriya’s liability under all of its policies. The
reinsurance contract contained a customary ‘no third party beneficiary’ clause, which
provided that only the reinsurer, cedent and the consumer group had rights under the
reinsurance contract. The reinsurer and the cedent also entered into an administrative
services agreement that required the reinsurer at its expense to provide all services of
administration for all of the cedents policies, including actuarial, underwriting, compliance,
legal, accounting, issuance etc,. When the cedent denied the certificate holder’s claim, the
certificate holder sued the cedent and the reinsurer. The reinsurer moved for summary

78
judgment, dismissing the action on the ground that there was no contractual relationship
with the certificate holder upon which liability could be established against the reinsurer.
Discuss the implications.
SOLUTION:
Cedent – Janapriya – could not delegate its duty to its insured and the reinsurance agreements
provided that they do not benefit third parties like the certificate holders. It is important to note
that the reinsurer is really the certificate holder’s insurer. A reasonable conclusion might be
drawn from the administrative services agreement that the cedent was simply fronting for the
reinsurer in the provision of health and accident insurance by the reinsurer to the public. The
agreement suggested, realistically, that all that the cedent was doing was providing insurance
policy and certificate forms bearing its name and address , and perhaps a sales organization,
but that the reinsurer was responsible for and did perform all other aspects of insurance
undertaken in the cedent’s name. Hence, the certificate holder’s claim is justified and the
reinsurer’s motion for summary judgment should be denied.

79
CHAPTER 12

REINSURANCE REGULATIONS

SECTION A

1. One of the following is not true:

a. The Dept. of Trade, UK requires the primary insurers to annually report the amount of
premium payable to each reinsurer.

b. In the US, reinsurers have to keep solvency margin almost along the lines prescribed for
primary insurers.

c. In US, the reinsurers have to file financial statements with State Regulatory Authorities
quarterly.

d. Pricing is not directly regulated.

e. None of the above*

2. one of the following is not true:

a. The regulation by means of an insolvency clause provides that the insolvency of the
primary insurer will not affect the liability of the reinsurer.

b. The reinsurer will make payment to the liquidator or the receiver of the insolvent primary
insurer for the benefit of their creditors.

c. The risk of insolvency of the reinsurance broker is passed on to the reinsurer so any
default by the reinsurance broker will be made good by the reinsurer.

d. Reinsurance broker is an agent of primary insurer.*

e. None of the above.

3. The intermediary must make full written disclosure of:

a. Any control over the broker by a reinsurer.

b. Any control of a reinsurer by the intermediary.

c. Any retrocessions of the subject business placed by the intermediary

d. Commissions earned or to be earned on the business;

e. None of the above*

4. Records of all transactions must be retained for at least------years after the expiration of all
reinsurance contracts.

80
a. 5

b. 7

c. 10*

d. 12

e. None of the above

5. In India the following authority administers regulations on insurance industry.

a. Ministry of Finance, Union Govt.

b. RBI

c. IRDA*

d. PFRDA

e. SEBI

6. One of the following is not an objective of reinsurance regulations in India.

a. to maximize the retention within the country.

b. Develop adequate capacity’

c. Secure the best possible protection for the reinsurance costs incurred;

d. Simplify the administration of business

e. None of the above.*

7. Regulations relating to placement of reinsurance business outside India:

a. those reinsurers who have a period of the past 5 years enjoyed a rating of at least
BBB (with S&P)

b. Rating of any international agency equivalent of S&P

c. Placement can be done with reinsurance agency in USA.

d. Prior permission of IRDA should be obtained.*

e. None of the above.

8. Which of the following Acts are relevant to reinsurance cases?

i. Indian Contract Act.

ii. Indian Arbitration Act.

81
iii. Insurance Act.

a. only (i)

b. only (ii)

c. Both (i) and (ii)

d. Both (ii) and (iii)

e. All of the above.*

9. Every insurer shall cede such percentage of the sum insured on each policy for different
classes of insurance written in India to the Indian reinsurer. What is that percentage?

a. 15.

b. 20.*

c. 30.

d. 40.

e. None of the above.

10. The Indian reinsurance agency is:

a. EXIM bank.

b. ECGC.

c. GIC.*

d. LIC.

e. IRDA.

SECTION A (Answers)

1.e. 2.d 3.e. 4. c. 5. c. 6.e 7.d 8.e. 9.b.10.c

SECTION B

1.Examine reinsurance regulation scenario in UK and USA briefly.

ANS:

In the U.K., the Department of Trade requires the primary insurers to annually report on the
following:
a. The names and addresses of all reinsurers to whom business has been ceded during the
year;

82
b. Any connection (other than the reported reinsurance) between the primary insurer and any of
its reinsurers;
c. The amount of premium payable to each reinsurer; and
d. Any indebtedness of a reinsurer to the primary insurer at the end of the year.
In the United States, reinsurers as well as licensed alien reinsurers have to keep solvency margin
almost along the lines prescribed for primary insurers and must follow reserves, investment, capital
and surplus requirements and annually or sometimes every quarter file financial statements with
State regulatory authorities.
2. “Reinsurance intermediaries act in a fiduciary capacity for all funds received in their professional
capacity and must not mingle them with other funds in USA” Explain the role of intermediaries.
ANS:

Reinsurance intermediaries shall have written authorization from the insurers and reinsurers they
represent, spelling out the extent and limitations of their authority;
The written authority above must be made available to the primary insurers or reinsurers with
which the intermediary deals;
No licensed intermediary shall procure reinsurance from an unlicensed reinsurer unless the
reinsurer has appointed an agent for the service of process in New York;
The intermediary must make full written disclosure of
Any control over the broker by a reinsurer,
Any control of a reinsurer by the intermediary,
Any retrocessions of the subject business placed by the intermediary, and
Commissions earned or to be earned on the business;

3.Briefly explain reinsurance regulations in India.


ANS:
In India, the Insurance Regulatory and Development Authority has prescribed regulations for the
reinsurance sector for general as well as life insurance.
The IRDA (General Insurance-Reinsurance) Regulations, 2000 are as follows.
The first chapter defines such special terms as cession, facultative, Indian reinsurer, pool,
retrocession, retention, and treaty.
The second chapter describes the procedure to be followed for reinsurance arrangements Clause
3-1 states the objectives of a reinsurance program as
To maximize retention within the country;
Develop adequate capacity;
Secure the best possible protection for the reinsurance costs incurred;

83
Simplify the administration of business.
4.State important provisions of Clause 3 – IRDA Regulations on reinsurance.
ANS:
Clause 3-2:Every insurer shall maintain the maximum possible retention commensurate with its
financial strength and volume of business.
Clause 3-3:Every insurer shall cede such percentage of the sum insured on each policy for
different classes of insurance written in India to the Indian reinsurer.
Clause 3-4:The reinsurance program of every insurer shall commence from the beginning of every
financial year and every insurer shall submit to the Authority, his reinsurance programs for the
forthcoming year.
Clause 3-5:Within 30 days of the commencement of the financial year, every insurer shall file with
the Authority a photocopy of every reinsurance treaty slip.
Clause 3-6:The Authority may call for further information or explanations in respect of the
reinsurance program of an insurer and may issue necessary directions.
Clause 3-7:Insurers shall place their reinsurance business outside India with only those reinsurers
who have over a period of the past five years counting from the year preceding for which the
business has to be placed, enjoyed a rating of at least BBB (with Standard & Poor) or equivalent
rating of any other international rating agency. Placements with other reinsurers shall require the
approval of the Authority.
Clause 3-8:The Indian reinsurer shall organize domestic pools for reinsurance surpluses in fire,
marine hull and other classes in consultation with all insurers on basis, limits, and terms which are
fair to all insurers.;
Clause 3-10:Every insurer shall offer an opportunity to other Indian insurers including the Indian
reinsurer to participate in its facultative and treaty surpluses before placement of such cessions
outside India.

SECTION C (case studies)

1.General Insurance Corporation of India has been well adapting itself to the changing reforms
scenario. To focus itself on the reinsurance market both domestic and international, it has taken
various decisions to support its new corporate vision. In Jan 2004, GIC has decided to exit its
mutual fund arm, GIC Mutual Fund, so as to focus on core reinsurance operations. The fund had
been constantly underperforming for the last few years. In 2002-03 there has been whopping
increase in the foreign inward reinsurance premia at Rs 600 crores. This increase has pushed the
total reinsurance premium to over Rs3800 crores. The Indian reinsurer, is willing to write more risks
in the domestic market. The underwriting losses fell below the Rs 500 core-mark. Though the
severe drought, took its toll on GIC’s underwriting with agricultural losses zooming to Rs 400 crores
in 2002-03. the claims ratio reduced during the year from 94 to 86%. Though the quantum of
foreign inward premium is low in the total premium income, the increase in its share over the last

84
one year is significant. In 2002-3, the share of foreign premium has been over 15% compared to
just 6% in the previous year.

International Credit rating agency, AM Best, has given “A (Excellent)” rating to the corporation
indicating its financial strength. The rating reflects not only the Corporations’ excellent financial
position and conservative investment portfolio but also recognizes its leading position in the global
market.

Discuss the growth opportunities and business of GIC in the light of latest global changes.

SOLUTION:

Today, India stands as a country whose economy is getting increasingly integrated with the world
economy. The large and growing market, developing infrastructure, sophisticated financial sector,
stable polity and strong economic outlook make India an attractive destination, both, for business
travelers and tourists.

GIC of India (GIC), the Indian Reinsurer caters to the reinsurance needs of the country and also
underwrites risks globally. GIC’s support to the Indian market comes in the form of obligatory
cessions, company surplus treaties, market surplus treaties, Excess of Loss (XOL) protection to
direct writing companies and facultative acceptances. GIC is one of the reputed underwriters for
treaty and facultative reinsurance in the Afro- Asian region.

As a sole reinsurer in the domestic reinsurance market, GIC provides reinsurance to the direct
general insurance companies in the Indian market. GIC receives statutory cession of 20% on each
and every policy subject to certain limits. It leads many of domestic companies’ treaty programmes
and facultative placements. GIC’s capacity for each class of business on Treaty and Facultative basis
for domestic business is given in the following table.

Class Capacity
Property INR 15000 mln. Any one risk
Marine Hull - INR 2000 mln. any one vessel
Cargo INR 2500 mln. Any one policy/sending
Crude Oil INR 3500 mln. any one policy/sending
Engineering INR 15000 mln. Sum Insured
Miscellaneous INR 3500 mln. Sum Insured
Motor Unlimited

Class Capacity
Aviation-Hull/Spares- Rotor Wing INR 20.50 mln. per aircraft

85
Aviation-Hull/Spares- -Fixed Wing INR 50 mln. per aircraft
Liability INR 100 mln. per occurrence
Aviation Treaty – Hull INR 500 mln.
Spares INR 250 mln.
Liability INR 1000 mln.

GIC declared a bonus in the ratio of 1:1 and the equity capital has grown to Rs. 430 crores.
Dividend @ 15% amounting to Rs. 64.50 crores was declared on the enhanced capital. The
income from investments amounted to Rs. 1398.61 crores in 2004-05. GIC booked a profit before
tax of Rs. 800.08 crores and profit after tax of Rs. 200.02 crores.

The Net Premium Income during 2004-05 amounted to Rs. 4613.87 crores representing a growth
of 10.83%. Keeping in line with the philosophy of utilizing Indian capacity optimally and maximizing
retentions, the net retention ratio stood higher at 90.09% in 2004-05 as against 89.7% in 2003-04.
Net Incurred claims in 2004-05 were to the tune of Rs. 3702.80 crores.

Total Investments as on 31.03.2005 amounted to Rs. 15434.31 crores, Total Assets increased to
Rs. 19551.84 crores and Total Funds as on 31.03.2005 stood at Rs. 11767.69 crores. On the
strength of Indian business and with focused marketing efforts in Afro-Asian region, GIC aims at
positioning itself within the top 20 Global Reinsurance in the years to come.

The ratings of GIC reflect its leading business profile in the Indian reinsurance market and an
expected improvement in underwriting performance. A.M. Best believes GIC will retain its leading
position within its domestic market despite an expected decline in domestic premium over the
medium term with the likely cessation of compulsory cessions from which it currently benefits as
India’s national reinsurer (currently 20% cession from domestic insurers). Furthermore, A.M. Best
expects GIC to increase its acceptance of international emerging market business through its
representative offices in London, Moscow and Dubai.

86
SECTION 13

REINSURANCE MARKETS

SECTION A

1.It is an association of reinsurers banded together to underwrite reinsurance jointly.

a. Lloyd’s Club.

b. Reinsurance Pool

c. Society of Reinsurers

d. World Re Federation.

e. None of the above.

2.one of the following is not true;

a. Some reinsurance pools write reinsurance for only members.

b. Industrial insurers may form a pool to write specific risks.

c. Specialized pools are also formed for energy insurance.

d. A&B

e. None of the above

3.Lloyd’s was originally developed as

a. Center for Marine insurance.

b. Center for Aviation insurance.

c. Center for General insurance.

d. Center for Life insurance.

e. None of the above.

4.This is the largest reinsurance company of the world.

a. Munich Re

b. Swiss Re

c. Employers Re

d. American Re

87
e. None of the above.

5.Most of the top reinsurers are located in;

a. Switzerland

b. Germany

c. United States of America

d. Britain

e. Japan.

6. One of the following is not true:

a. In the modern days reciprocal reinsurance is not widely practiced

b. The reinsurance market in London is constituted of Lloyds.

c. The percentage commission paid by the reinsurers to the reinsurance brokers is relatively
high.

d. B&C

e. None of the above.

7.AXA Re is associated with which country?

a. Netherlands.

b. Germany

c. USA.

d. France.

e. Japan

8. Generally reinsurance brokers prefer this method of reinsurance.

a. Facultative reinsurance

b. Excess of Loss Reinsurance.

c. Treaty Reinsurance

d. Quota share cover.

e. None of the above.

9. Identify the factor which is not contributing to the success of reinsurance market:

88
a. Availability of knowledge capital.
b. Matured financial markets.
c. Infrastructure facilities .
d. Domestic markets.
e. Interest rates.

10. After nationalization of the insurance industry, five companies have taken care of
general insurance needs. Identify the odd man out.

a. General Insurance Corporation of India.


b. Life Insurance Corporation of India
c. Oriental Insurance Company.
d. United India Insurance Company.
e. National Insurance Company.

11. The setting up of a Regional Reinsurance Corporation depends on certain common


features. One of the following is not true:

a. Countries must have common physical boundaries.


b. Countries must have well-developed communication between member countries.
c. Countries must have common currency.
d. Must share common customs, ethnic identity and language.
e. None of the above.

12. The insurance companies formed by large commercial or industrial establishments,


essentially to care of their own insurance needs:

a. In-house Insurance.
b. Self-Insurance.
c. Captives
d. Private Agencies.
e. None of the above.

13. Tax concessions by the local governments are a major attraction to:

a. In-house Insurance.
b. Self-Insurance.
c. Captives
d. Private Agencies.
e. None of the above.

14. This provides coverage to the single owners who hold the company.

a. Pure Captives.
b. Associate Captives.
c. Beneficiary Captives.
d. In-house Enterprises.

89
e. None of the above.

15.One of the following is not a benefit of captives.

a. Operating costs are reduced.


b. There is improved cash flow.
c. There is a direct reach to wholesale reinsurance markets.
d. There is greater control over claims.
e. None of the above

SECTION A (Answers)

1.b. 2.e 3.a. 4.a. 5. c. 6.c 7.d 8.c. 9.e.10.b. 11.c 12. c. 13.c. 14.a. 15.e.

SECTION B

1.Mention any top ten reinsurance brokers in the world.

ANS:

Company Country of Domicile


Munich Re Germany
Swiss Re Switzerland
Employers Re United States
General re/Cologne Re Group United States
Assicurazioni Generali S.p.A. Italy
Hanover Re Group Germany
Lincoln National Re United States
Gerling Global Re Group Germany
SCOR Group France
Tokio Marine & Fire Ins. Group Japan

2.Explain the role of reinsurance brokers.

ANS:

Just as there may be brokers for primary insurers who act as intermediaries between the insured
and the insurer, there are reinsurance brokers who are go-betweens to primary and reinsurers. The
percentage commission paid by the reinsurers to the reinsurance brokers is relatively small,
compared to the commission paid to the insurance brokers and is sometimes as low as one
percent of the reinsurance premium.

90
When there are reinsurance brokers, the premium payments and loss payments as well as
premium refunds pass through them. When primary insurers do not have expertise to place
reinsurance directly, they need the services of the reinsurance brokers. Large reinsurers also use
reinsurance brokers as a matter of course. However, if primary insurers go direct to reinsurers,
they may be able to reduce the reinsurance cost to some extent.
Although reinsurance brokers obtain their commission from the reinsurers, they have a duty to
observe the principle of utmost good faith, which means they must reveal to the reinsurers all
material facts concerning the risks, which they obtained from the primary insurers. The market
share of the reinsurers combined in the United States is estimated at 75%, which shows the
predominance of the reinsurance brokers in the reinsurance market. Generally, reinsurance
brokers handle treaty reinsurances in preference to facultative reinsurance.

3.Outline the features of a successful reinsurance market:


ANS:
The features that exemplify the reinsurance market are as follows:
1. Stability factor.
2. Availability of Knowledge Capital.
3. Matured Financial Markets.
4. The Role of Economy.
5. The Arbitration Factor.
6. Infrastructure Facilities.
7. Domestic Markets.
8. The Location Advantage.
9. The Role of Foreign Reinsurers.

4. What is the Regional Reinsurance Corporation? Outline their role.


ANS:
Generally, a regional reinsurance corporation caters to the needs arising among a group of
neighboring countries. These corporations were proposed to be set up across the different
developing nations of the world.
The setting up of regional reinsurance mainly depends on certain common features, which the
member countries are bound to have due to the binding proximity with or to each other.
The regional reinsurance corporations have the liberty to choose their own market place to locate
their head quarters. The head quarters may be set up depending on the factors such as well-
developed accessibility, excellent communication facility, well-established commercial backup etc.
The backing of good banking system will enable the corporation to have a smooth functioning.

91
The regional reinsurance corporations end up with a good business in hand by involving
compulsory cessions from its members. The corporation can also get exposed to the risks arising
from the member nations. In order to avoid such happenings they get protection through
retrocession covers from other markets. The regional reinsurance corporations manage their
portfolio, which is spread over different countries through various retrocession covers they are:
• Proportional covers
• Non-proportional covers
• By trading outward covers through traders and generating inward covers.
• Non-reciprocal inward reinsurance accounts, a part of which may be retroceded back to the
member nations.
5.What is Captive? Explain various types of Captives.
ANS:
The insurance companies formed by large commercial or industrial establishments, essentially to
take care of their own insurance needs, are called captives. These captives play a major role in
fulfilling the insurance needs of the parent companies and help the money flow inside the periphery
of the business group.
The various kinds of captives can be branched out as follows:
Single Parent Captives:
Single parent captives are also called as ‘pure’ captives. This provides coverage to the single
owners who hold the company. A risk manager or financial officer at the parent company
monitors them.

Association captives:
An established association generally forms this captive. The coverage is provided to its
members. In this the ownership vests with the association or the individual members. The
financial expert at the association level looks after the operation or this responsibility is
outsourced to a management company, consultant or a broker.
Industry captives:
The industry captive is owned by industries with similar specific insurance problem. A board of
director is appointed by the shareholders to whom the company is required to report.
Agency captives:
An agent or group of agents owns this particular company. These are formed such that their
clients can participate in the programs.
Rent-A-Captives:
The risks of the members are insured. The investment income and underwriting profits are
returned back to the insureds. Under this the surplus of certain companies are given on rent in
order to establish a self-insurance program and not their own captive.

92
Protected cell companies:
These are special category of rent-a-captive since they shield their capital and surplus from other
renters in the captive until the rent-a-captive ‘s owner remains solvent.
6.What are benefits of Captives?
The corporations and groups who want to take a financial control and manage risks by
underwriting their own insurance than preferring to pay premiums to the third-party insurers
can opt for captives. Captive is nothing but a tool to such organizations.
The benefits of captives are as follows:
Provides insurance for certain exposures, which other insurance companies might not
provide.
Enables to retain the premiums within the group by the parent company.
Operating costs are reduced.
There is an improved cash flow.
There is an increase in coverage and capacity.
Better investment as well as investment income.
There is a direct reach to wholesale reinsurance markets.
There is flexibility in underwriting and funding.
There is a greater control over claims.
Availability of smaller deductibles for operating units.
There is an additional negotiating leverage with underwriters.
Availability of incentives for loss control.

SECTION C (Case Studies)

1. The weather has a substantial influence on the economy — from the risk of a rainy summer for
beer garden proprietors to the dependency of energy suppliers on the weather. Weather
derivatives offer a range of tools for these industries to hedge against previously uninsurable
weather risks.
Hedging against a cold summer

"When the temperature in Britain rises by 3°C, daily beer consumption increases by 10%", says the
UK Met Office in a study — a very telling example that shows how much a particular industry's
turnover can be affected by the weather.

Meteorological research institutes estimate that more than 80% of all economic activities are
weather-dependent. In other words, a huge variety of industries are involved — fashion companies,
breweries, construction firms, ice-cream producers and especially energy suppliers.

93
When national utility monopolies in the USA were turned into independent energy companies,
demand increased for protection against the impact of the weather. Instruments for hedging
weather risks were sought, and what more obvious idea than to use the tools of the capital market?
Explain how Munich Re is offering weather derivatives.

SOLUTION:

Objective of weather derivatives

A weather derivative differs from a customary derivative in that it is not based on tradable reference
items such as shares, share price indices, bonds or exchange rates but on predetermined index
data, such as the temperature. It follows that the purpose of a weather derivative is to hedge
volume risks rather than price risks, since a rising or falling turnover need not affect the price.

In a cold summer, for instance, it is not usual for the price of ice-cream to suddenly fall — the ice-
cream industry's decrease in turnover is due to the lower sales volume, then, not to a price
reduction. In these cases, weather derivatives provide volume compensation.

A similar situation exists in future-oriented fields like renewable energy: particularly in the first few
years, when high investment costs have to be recouped, a lack of wind or too little solar irradiation
might reduce the volume of electricity generated to such an extent that the financing of the venture
is jeopardised. Weather derivatives can then provide important balance-sheet protection.

From single trigger to multiple trigger

Of course there are also many industries where the weather has an impact on both the volume and
the price, such as the oil industry. If a winter is warmer than average, people heat less — there is a
fall in the sales volume and the price of fuel oil.

In such cases, "cross-hedging" is recommended, a mixture of weather derivative and classic


commodity derivative, which also hedges against a decrease in turnover as a result of price
reductions.

Within the framework of integrated risk management, Munich Re offers multiple-trigger models that
cover several risks simultaneously.

What constitutes bad weather?

The most widespread trigger used for weather derivatives is the temperature. However, other
triggers are agreed on sometimes as well, such as certain amounts of precipitation, wind, sunshine
or snowfall.

Weather derivatives are usually concluded for one season only, but we also offer our clients
contracts that are valid for a shorter or longer period.

94
Munich Re's know-how

Optimally designing and deploying weather derivatives for clients requires geoscientific expertise
combined with know-how in statistical methodology and knowledge of the capital markets.

Munich Re has been offering extensive weather and climate consultancy services for over 30 years
and assess over 500 weather risks a year. Besides this, they have a sufficiently diversified portfolio
which they actively manage, thus meeting all the requirements for successful business with
weather derivatives.

2. The insurance industry has a number of critical cogs that allow it to work properly. One of the
most important cogs in the mechanism is the reinsurance market. And like the primary insurance
market, the last few years have proven to be quite challenging for reinsurers. A long list of issues
has shaped the current reinsurance marketplace. Among the more significant contributing factors
were: 9/11, a prolonged soft insurance market, low returns on investment income and the need for
general reserve strengthening.

Current state of the market

Financial results for the reinsurance market are similar to those of the general insurance market.
For example, financial results for the first quarter of 2004 show a marked improvement in
profitability. The combined ratio for the reinsurance industry as reported by the Reinsurance
Association of America is 94, better than last year's healthy 96.4. This continues a trend of
steadily improving results over the past few years. Despite this rosy present, many experts are
still concerned about the long-term prospects for the reinsurance industry.

Standard & Poor's is one of the more vocal critics of reinsurers'future financial conditions. It
continues to report that the outlook for the reinsurance industry remains negative. This basically
means that ratings downgrades for U.S. reinsurers are expected to exceed upgrades in the
current year. Discuss.

SOLUTION:

Poor financial results over the past few years have required a number of changes in the way
reinsurers approach their core business, as well as operational aspects in their underwriting.
Following is a review of some of the more interesting changes, as well as a report on several key
trends that will have a profound impact on the industry in the future.

Risk securitization

It's obvious that the reinsurance market is undergoing rapid changes in the nature of its core
business practices. For the most part, this has been caused by the convergence of the financial
markets. Leading the change in core business practices is the securitization of risk.

A recent study by Guy Carpenter found that in 2003 the catastrophe bond (CAT bond) market
had significant growth. The primary purpose of CAT bonds is to provide a mechanism for

95
reinsurers to raise capital to cover catastrophic claims. An additional purpose, however, is to offer
a portfolio diversification strategy to institutional investors. Up to this point, the most popular CAT
bonds were associated with property risk such as windstorm and earthquake exposures in the
United States, but now there is growing interest in other types of applications. Current
possibilities include such diverse areas as securitizing life insurance portfolios and non-property
exposures such as workers compensation and terrorism. A number of other innovative
applications are also being reviewed for financial viability.

While many in the insurance community see the value of the CAT bond approach, much of the
current movement is being driven by investors looking for higher yield investment vehicles.
Among other reasons for the increasing attention are:

* Investors are becoming more comfortable with the securitization approach.

* The structure of securitizations is becoming more standardized.

* Time and cost involved in completing a securitization have been greatly reduced.

Both Guy Carpenter and Swiss Re predict significant growth in securitizations near term. As
innovation and consumer acceptance continues in this market segment, one would expect to see
this trend of the gradual blending of the financial service products to accelerate.

At this point, most industry experts and rating agencies are painfully aware of the potential
problem associated with questionable recoverables and are implementing reductions in their
listings of future recoverables. As a result, some companies are taking more of the risk internally,
while seeking to place business with higher rated reinsurance companies when they do choose
to use that market. An additional failsafe mechanism is a National Association of Insurance
Commissioners (NAIC)- USA rule that requires insurers to reduce reinsurance recoverables by
20% for those that are overdue by more than 90 days. Accordingly, there has been an overall
trend to move the reinsurance business to more highly rated reinsurers. This, of course, has the
predictable effect of placing premium pricing on financially sound reinsurers. It is expected that
this trend will persist over the next few years as the industry continues to struggle with adequate
amounts of capital.

96
CHAPTER 14

INWARD REINSURANCE

SECTION - A

1. One of the following is not true:

a. Reinsurance business accepted by an insurer is inward reinsurance business


b. Reinsurance business accepted by a reinsurer, is inward reinsurance business
c. It is opposed to insurance business ceded to another insurer.
d. A&B
e. None of the above.

2.Why companies go for inward reinsurance business?

a. To increase the gross premium and net retained premium


b. To achieve a lower expense ratio by maintaining the volume of premium income as ceding
reduces the premium income.
c. To obtain a better and wider spread of business
d. All of the above.
e. None of the above.

3.One of the following is not true:

a. A company has to devise it’s corporate strategy for its underwriting policy.
b. It can write lines for its net account.
c. It can write larger shares.
d. It can create a retrocession treaty to take care of surplus over its net retention.
e. None of the above.

4.This consideration need not be kept in mind while finalizing inward reinsurance;

a. Treaty or Facultative reinsurance


b. Territorial scope.
c. Direct or Brokers.
d. Class of business
e. None of the above

5.One of the following is not true:

a. Facultative reinsurance involves more administrative work.


b. Treaty is less expensive but requires thorough knowledge of the market.
c. If the company wants a greater geographical spread then it should underwrite foreign
business.
d. If the company has experienced staff direct business can be solicited.
e. None of the above.

97
SECTION A (Answers)

1.e. 2.d 3.e. 4. e. 5.e

SECTION B

1.Why do companies go for inward reinsurance?

ANS: Some of the reasons why companies go for inward reinsurance are as follows:

• to increase the gross premium and net retained premium


• to achieve a lower expense ratio by maintaining the volume of premium income as ceding
reduces the premium income
• to obtain a better and wider spread of business
• to counteract the drain of foreign exchange caused by ceding of premium

• to earn an investment income which may be derived from the cash flow.

2.What are the considerations a company should keep in mind while finalizing its inward program
of reinsurance?

ANS:

Some considerations the company should keep in mind while finalizing its inward programme for
the year are as follows
• Treaty or facultative –facultative involves more administrative work as each offer will be
scrutinized. Treaty is less expensive but it requires a thorough knowledge of the market and
treaty clauses.
• Territorial scope-if the company wants a greater geographical spread then it should
underwrite foreign business keeping in view the political and economic conditions of the
country.
• Direct or brokers – if the company has experienced staff direct business can be solicited.
However this will involve travel expenses to procure business. So initially it is better to place
business through a broker.
• Class of business- the company should decide whether it wants to underwrite property
business, which is on annual basis or casualty business.
• Acceptance limits-keeping in mind the financial standing and premium income of the company
the acceptance limit should be large enough to make it attractive for the brokers and ceding
companies to offer business.

98
CHAPTER 15

REINSURANCE PRACTICE
SECTION A

1.Which of the following statements is true?

a. A good reinsurance program can be executed with assistance from reinsurers, brokers
and consultants.
b. A reinsurance plan must take into account the primary insurer’s needs.
c. The primary insurers must continue to be solvent.
d. A&B
e. A,B & C.

2.one of the following is not a factor determining the reinsurance needs.


a. Kinds of insurance written
b. Volume of insurance written
c. Available financial resources.
d. Business volume of client
e. None of the above

3. Law of large numbers is not applicable in one of the following cases.


a. Motor reinsurance
b. Catastrophes
c. Marine insurance
d. Life insurance
e. None of the above.
4. The important factor in setting of the retention

a. Stability
b. Liquidity of investment
c. Amount of relief needed
d. Growth plans.
e. None of the above.

5.The principal purpose of an excess of loss treaty is

a. stabilize loss exposures


b. providing large line capacity
c. stabilize earnings
d. A&B
e. A,B&C

6 Setting reinsurance limits depends on

a. Company considerations
b. Cost considerations

99
c. Financial strength of the reinsurer
d. Financial strength of the client
e. None of the above.
7.While setting the reinsurance limits this factor is not considered
a. Volume of the premium
b. Premium loading
c. Reinsurance cost
d. All the above.
e. None of the above

8.One of the following is more vulnerable to loss ratio fluctuation.

a. National level primary insurer


b. Regional insurer
c. Marine insurer
d. Life Insurer
e. None of the above.

9. The reinsurance cost includes

a. Operating expenses
b. Premium paid to reinsurer
c. Losses recovered / to be recovered
d. A&B
e. B&C

10. One of the following is not true:

a. Transfer of assets results in loss of investment income to the primary insurer.


b. Such a loss of investment income may be greater under pro-rata treaty.
c. Reinsurance premium for a pro-rata treaty is usually lower.
d. Loss of investment income may also become an additional cost of reinsurance.
e. None of the above.

11. Negotiations for reinsurance depend on several factors. One of the following is not one among
them.

a. The nature of the primary insurer


b. The reinsurer
c. Kind of reinsurance transacted
d. Interest rates
e. None of the above.
12. In facultative reinsurance negotiations, one of the following is important

a. the reinsurer
b. the primary insurer
c. details of individual loss exposures

100
d. underwriting operations.
e. None of the above.

13. One of the following is not true in assessing the underwriting policy of a primary insurer.

a. The classes of business the primary insurer is writing.


b. How satisfactory are the primary insurer’s underwriting guidelines?
c. Are their gross line limits and net line limits in keeping with their financial strength?
d. The primary insurers’ rates are not adequate for the risks covered under the treaty.
e. None of the above.

14. These are the commissions involved in the reinsurance transaction.

a. Ceding Commission
b. Brokerage Commission
c. Settlement Commission
d. A,B&C
e. A&B

15. The premium registered in the books of an insurer at the time a policy is issued, is called;

a. Earned premium
b. Unearned premium
c. Written premium
d. Registered premium
e. None of the above

SECTION A (Answers)

1.e. 2.d 3.b. 4.c. 5. d. 6.b 7.b 8.b 9.e.10.c 11. d. 12. c. 13.d. 14.e. 15.c.

SECTION B

1. What are the factors determining the reinsurance needs of a primary insurer?

ANS:

The important factors are:


Kinds of insurance written
Exposures subject to catastrophic loss
Volume of insurance written
Available financial resources
Stability and liquidity of investment portfolio, and
Growth plans.

101
2. Explain the concept of ‘Setting Retentions’.

ANS:
The choice of retention depends on the type of treaty, which in turn depends on the needs of the
primary insurer. The setting of retention varies depending on the type of treaty. In other words the
basic reason for choosing one type of treaty in preference to another is supported by the following
example. For example, if the primary insurer prefers a pro-rata treaty when compared to an excess
treaty the reason could be that a pro-rata treaty provides surplus relief. Therefore, the important
factor in the setting of retention must be the amount of relief needed. The amount of surplus relief
received will be a function of the percentage of premiums ceded and the percentage ceding
commission received.

3. Explain the process of setting reinsurance limits.


ANS:
Setting the reinsurance limits depends on cost considerations since reinsurance costs increase in
direct proportion to reinsurance limits, keeping the retention constant. However, the treaty
reinsurance costs must be weighed against other recurring costs in facultative placements such as
the premium, administrative expense and inconvenience and uncertainty associated with
facultative reinsurance. However, while setting the reinsurance limits only the volume of the
premium is considered and not the premium loading. Limit Setting for a Catastrophe treaty is even
more difficult in practice since one cannot predict a large loss merely based on historical records.
Therefore, in reinsuring catastrophes, concentration of loss exposures must be carefully analyzed.
In the exercise of setting the reinsurance limit, the terms of the several treaties must be compared
and the limits can be kept flexible. For example, the limit for an aggregate excess treaty can be
lowered if adequate catastrophe reinsurance is carried. Again the limit of a catastrophe can be
lower if it applies only to the retention of the primary insurer after recoveries from pro-rata
reinsurance, rather than to the direct losses.

4. Cost of Reinsurance is an important element in finalizing reinsurance deal. Explain the cost
of reinsurance.
ANS:

The reinsurance cost includes the premium paid to the reinsurer and the losses recovered or to be
recovered under the reinsurance agreement. A primary insurer should pay its own losses and the
reinsurer’s expenses and profit under any treaty, if the treaty is continued over a fairly long period.
That is why the amount included in the premium for the reinsurer’s expenses and profit is an
important factor in assessing the reinsurance cost.
Loss of investment income may be greater under a pro-rata treaty than under the excess treaty
since the reinsurance premium for a pro-rata treaty is usually greater. Thus, the loss of investment
income may also become an additional cost of reinsurance. The cost of administering the
reinsurance program varies depending upon the type of reinsurance. For instance, since facultative
placements are individual and separate, the cost of administration in these cases is greater than in

102
the case of treaties. Similarly pro-rata treaties cost more to administer than excess treaties.Finally,
the profit or loss on insurance assumed under reciprocal arrangement must also form part of the
reinsurance cost.
5. What are the typical questions to be considered in assessing the underwriting policy of a
primary insurer?
ANS:
The classes of business the primary insurer is writing;
If they are primarily concentrating on personal lines, commercial, industrial or others;
Their geographic area of operation;
How satisfactory are the primary insurer’s underwriting guidelines?
Are their gross line limits and net line limits in keeping with their financial strength?
Are the primary insurer’s loss control and loss adjustment practices adequate for the classes of
business written?
Have the primary insurer’s underwriting results been satisfactory in the lines covered by the
proposed reinsurance treaty?
Does the primary insurer anticipate any substantial changes in its management, marketing or
underwriting practices?
Are the primary insurer’s rates adequate for the risks covered under the treaty?

SECTION C (case studies)


Himalayan Insurance company is a large insurance company. It specializes in personal lines
insurance, motor insurance and mortgage insurance. Its annual direct premium volume is Rs 2000
crores. Its business is spread all over the country, with the proportion of its business in any one
state being approximately the same as that state’s proportion of the nation’s population. Its policy
holders surplus is Rs 1200 crores. Its investment portfolio consists mostly of Government Bonds
and high-grade industrial and utility bonds. The insurance company normally holds about Rs 500
crores in treasury bills, high rated Commercial Paper and other short term assets. Investment in
equity stocks account for only 5 percent of its invested assets and less than 10 percent of policy
holders’ surplus. The insurance company has shown an operating profit every year for the last one
decade. An analysis of the geographic spread of its business exposures indicates that losses in
excess of Rs 500 crores in any one catastrophe are very unlikely.
Examine the reinsurance planning process of Himalayan Insurance Company.

SOLUTION:
After reviewing all of the available data, it is suggested that the Himalayan Insurance Company
does not require any reinsurance. Its large premium volume, wide spread of small individual risks,
strong surplus position and stable, liquid investment portfolio would enable it to cope with any loss
ratio variation it might reasonably expect to occur, including catastrophes.

103
2.Dharma Insurance company has annual direct written premiums of Rs 100 crores and policy
holders’ surplus of Rs 50 crores. It has averaged 10 percent annual growth over the past decade
and expects to continue to grow at about the same rate. Its combined ratio has exceeded 100
percent each year for the last decade, averaging 102 percent for the period. It has reported an
operating profit each year for the last decade, but the profit has been small in some years. A high
rated portfolio of bonds and other fixed-income securities accounts for about 95 per cent of its
invested assets. The balance consists of carefully selected preferred and equity stocks.

The Dharma writes commercial motor insurance, general liability, fire and allied lines of insurance.
It writes small to medium- sized commercial and industrial risks. A survey of its outstanding policies
showed that about 95 percent of its liability policies had occurrence limits of Rs1.0 crore and less,
and about 95 percent of other policies proved coverage of Rs 1.5 crore or less. Management has
decided to set its treaty limits to cover those amounts and to depend on facultative reinsurance for
the policies with greater limits. The Dharma does not need surplus relief, so no pro rata
reinsurance will be purchased. Discuss.

SOLUTION:
Per risk or per policy excess treaties will be purchased for both liability and property insurance.
Management may decide that the insurer can afford to assume individual losses upto 0.5 per cent
of direct written premium, or 1.0 per cent of policy holders surplus. Consequently the retention
under both the liability and other treaties will be set at Rs 50 lacs per loss. Treaty limits will be Rs
50 lacs for liability and Rs 1.0 crore for mortgage losses. Thus, the primary insurer’s retention and
the treaty limits will fully cover about 95 percent of all policies issued.
The properties insured by the primary insurer are spread widely across the country, but several
areas have high concentrations of values subject to catastrophe losses. The worst tragedy can
occur in coastal areas where losses could go up to Rs 5.0 crore. A catastrophe treaty with a limit of
Rs 5.0 crore and a retention of Rs 50 lacs will be purchased to cover the exposure.

104

Vous aimerez peut-être aussi